EXAM 2

Lakukan tugas rumah & ujian kamu dengan baik sekarang menggunakan Quizwiz!

The most common cause of decreased variability in the fetal heart rate (FHR) that lasts 30 minutes or less is: a. Altered cerebral blood flow. b. Fetal hypoxemia. c. Umbilical cord compression. d. Fetal sleep cycles.

ANS: D A temporary decrease in variability can occur when the fetus is in a sleep state. These sleep states do not usually last longer than 30 minutes. Altered fetal cerebral blood flow would result in early decelerations in the FHR. Fetal hypoxemia would be evidenced by tachycardia initially and then bradycardia. A persistent decrease or loss of FHR variability may be seen. Umbilical cord compression would result in variable decelerations in the FHR.

In order to care for obstetric patients adequately, the nurse understands that labor contractions facilitate cervical dilation by: a. Contracting the lower uterine segment. b. Enlarging the internal size of the uterus. c. Promoting blood flow to the cervix. d. Pulling the cervix over the fetus and amniotic sac.

ANS: D Effective uterine contractions pull the cervix upward at the same time that the fetus and amniotic sac are pushed downward. The contractions are stronger at the fundus. The internal size becomes smaller with the contractions; this helps to push the fetus down. Blood flow decreases to the uterus during a contraction.

A nurse may be called on to stimulate the fetal scalp: a. As part of fetal scalp blood sampling. b. In response to tocolysis. c. In preparation for fetal oxygen saturation monitoring. d. To elicit an acceleration in the fetal heart rate (FHR).

ANS: D The scalp can be stimulated using digital pressure during a vaginal examination. Fetal scalp blood sampling involves swabbing the scalp with disinfectant before a sample is collected. The nurse would stimulate the fetal scalp to elicit an acceleration of the FHR. Tocolysis is relaxation of the uterus. Fetal oxygen saturation monitoring involves the insertion of a sensor.

A group of nursing students are reviewing the various medications used for pain relief during labor. The students demonstrate understanding of the information when they identify which agent as the most commonly used opioid? A) Butorphanol B) Nalbuphine C) Fentanyl D) Meperidine

Ans. D Meperidine (dermol) Of all of the synthetic opioids (butorphanol [Stadol], nalbuphine [Nubain], fentanyl [Sublimaze], and meperidine [Demerol]), meperidine is the most commonly used opioid for the management of pain during labor.(pg. 464)

The nurse is reviewing the monitoring strip of a woman in labor who is experiencing a contraction. The nurse notes the time the contraction takes from its onset to reach its highest intensity. The nurse interprets this time as which of the following? A) Increment B) Acme C) Peak D) Decrement

Ans: A Feedback: Each contraction has three phases: increment or the buildup of the contraction; acme or the peak or highest intensity; and the decrement or relaxation of the uterine muscle fibers. The time from the onset to the highest intensity corresponds to the increment.

The nurse is reviewing the medical record of a woman in labor and notes that the fetal position is documented as LSA. The nurse interprets this information as indicating which of the following is the presenting part? A) Occiput B) Face C) Buttocks D) Shoulder

Ans: C Feedback: The second letter denotes the presenting part which in this case is "S" or the sacrum or buttocks. The letter "O" would denote the occiput or vertex presentation. The letter "M" would denote the mentum (chin) or face presentation. The letter "A" would denote the acromion or shoulder presentation.

Which of the following factors would the nurse suspect as predisposing a client to placenta previa? A. Multiple gestation B. Uterine anomalies C. Abdominal trauma D. Renal or vascular disease

A. Multiple gestation is one of the predisposing factors that may cause placenta previa. Uterine anomalies abdominal trauma, and renal or vascular disease may predispose a client to abruptio placentae.

9. A nurse is caring for a laboring woman who is in transition. Which of the following signs/symptoms would indicate that the woman is progressing into the second stage of labor? Select all that apply. (3) 1. Bulging perineum. 2. Increased bloody show. 3. Spontaneous rupture of the membranes. 4. Uncontrollable urge to push. 5. Inability to breathe through contractions.

1, 2, and 4 are correct. As the fetal head descends through a fully dilated cervix, the perineum begins to bulge, the bloody show increases, and the laboring woman usually feels a strong urge to push. 1. A bulging perineum indicates progression to the second stage of labor. 2. The bloody show increases as a woman enters the second stage of labor. 4. With a fully dilated cervix and bulging perineum, laboring women usually feel a strong urge to push.

Which maternal condition is considered a contraindication for the application of internal monitoring devices? a. Unruptured membranes b. Cervix dilated to 4 cm c. External monitors in current use d. Fetus with a known heart defect

ANS: A In order to apply internal monitoring devices, the membranes must be ruptured. Cervical dilation of 4 cm permits the insertion of fetal scalp electrodes and intrauterine catheter. The external monitor can be discontinued after the internal ones are applied. A compromised fetus should be monitored with the most accurate monitoring devices.

The nurse observes on the fetal monitor a pattern of a 15-beat increase in the fetal heart rate that lasts 15 to 20 seconds. What does this pattern indicate? a. A well-oxygenated fetus b. Compression of the umbilical cord c. Compression of the fetal head d. Uteroplacental insufficiency

ANS: A Accelerations in the fetal heart rate suggest that the fetus is well oxygenated.

The nurse providing care for the laboring woman should understand that amnioinfusion is used to treat: a. Variable decelerations. b. Late decelerations. c. Fetal bradycardia. d. Fetal tachycardia.

ANS: A Amnioinfusion is used during labor either to dilute meconium-stained amniotic fluid or to supplement the amount of amniotic fluid to reduce the severity of variable decelerations caused by cord compression. Amnioinfusion has no bearing on late decelerations, fetal bradycardia, or fetal tachycardia alterations in fetal heart rate (FHR) tracings

When a pregnant woman arrives at the labor suite, she tells the nurse that she wants to have an epidural for delivery. What is a contraindication to an epidural block? a. Abnormal clotting b. Previous cesarean delivery c. History of migraine headaches d. History of diabetes mellitus

ANS: A An epidural block is not used if a woman has abnormal blood clotting

How should the nurse intervene to relieve perineal bruising and edema following delivery? a. Place an ice pack on the area for 12 hours. b. Place a warm pack on the perineal area for 24 hours. c. Administer aspirin to relieve inflammation. d. Change the perineal pad frequently.

ANS: A An ice pack can be placed on the mothers perineum to reduce bruising and edema for 12 hours followed by a warm pack after the first 12 to 24 hours after delivery.

The nurse providing newborn stabilization must be aware that the primary side effect of maternal narcotic analgesia in the newborn is: a. Respiratory depression. b. Bradycardia. c. Acrocyanosis. d. Tachypnea.

ANS: A An infant delivered within 1 to 4 hours of maternal analgesic administration is at risk for respiratory depression from the sedative effects of the narcotic. Bradycardia is not the anticipated side effect of maternal analgesics. Acrocyanosis is an expected finding in a newborn and is not related to maternal analgesics. The infant who is having a side effect to maternal analgesics normally would have a decrease in respirations, not an increase.

48. An ultrasound report states, "The fetal head has entered the pelvic inlet." How should the nurse interpret this statement? 1. The fetus is full term. 2. The fetal head has entered the true pelvis. 3. The fetal lie is horizontal. 4. The fetus is in an extended attitude.

2. The inlet's boundaries are: the sacral promontory and the upper margins of the ilia, ischia, and the symphysis pubis. This is the entry into the true pelvis.

A low-risk 38-week gestation woman calls the labor unit and says, "I have to come to the hospital right now. I just saw pink streaks on the toilet tissue when I went to the bathroom. I'm bleeding." Which of the following responses should the nurse make first? 1. "Does it burn when you void?" 2. "You sound frightened." 3. "That is just the mucous plug." 4. "How much blood is there?"

2. The nurse is using reflection to acknowledge the client's concerns

Which occurrence is associated with cervical dilation and effacement? a. Bloody show b. False labor c. Lightening d. Bladder distention

ANS: A As the cervix begins to soften, dilate, and efface, expulsion of the mucous plug that sealed the cervix during pregnancy occurs. This causes rupture of small cervical capillaries. Cervical dilation and effacement do not occur with false labor. Lightening is the descent of the fetus toward the pelvic inlet before labor. Bladder distention occurs when the bladder is not emptied frequently. It may slow down the descent of the fetus during labor.

A gravid client, G3 P2002, was examined 5 minutes ago. Her cervix was 8 cm dilated and 90% effaced. She now states that she needs to move her bowels. Which of the following actions should the nurse perform first? 1. Offer the client the bedpan. 2. Evaluate the progress of labor. 3. Notify the physician. 4. Encourage the patient to push.

2. The nurse should first assess the progress of labor to see if the client has moved into the second stage of labor. TEST-TAKING TIP: The average length of transition in multiparas is 10 minutes. This client is therefore likely to have moved into the second stage of labor. The nurse's fi rst action, therefore, is to assess the progress of labor. If she is in second stage, the physician will be notifi ed and the client will be encouraged to push. If she is not yet in second stage, she should continue breathing with her contractions.

The nurse wishes to assess the variability of the fetal heart rate. Which of the following actions must the nurse perform at this time? 1. Place the client in the lateral recumbent position. 2. Carefully analyze the baseline data on the monitor tracing. 3. Administer oxygen to the mother via face mask. 4. Ask the mother to indicate when she feels fetal movement.

2. The variability of the fetal heart rate is Determined by analyzing the beat-to-beat fluctuations of the baseline rate.

With regard to nerve block analgesia and anesthesia, nurses should be aware that: a. Most local agents are related chemically to cocaine and end in the suffix -caine. b. Local perineal infiltration anesthesia is effective when epinephrine is added, but it can be injected only once. c. A pudendal nerve block is designed to relieve the pain from uterine contractions. d. A pudendal nerve block, if done correctly, does not significantly lessen the bearing-down reflex.

ANS: A Common agents include lidocaine and chloroprocaine. Injections can be repeated to prolong the anesthesia. A pudendal nerve block relieves pain in the vagina, vulva, and perineum but not the pain from uterine contractions, and it lessens or shuts down the bearing-down reflex.

A woman is experiencing back labor and complains of intense pain in her lower back. An effective relief measure would be to use: a. Counterpressure against the sacrum. b. Pant-blow (breaths and puffs) breathing techniques. c. Effleurage. d. Conscious relaxation or guided imagery.

ANS: A Counterpressure is steady pressure applied by a support person to the sacral area with the fist or heel of the hand. This technique helps the woman cope with the sensations of internal pressure and pain in the lower back. The pain management techniques of pant-blow, effleurage, and conscious relaxation or guided imagery are usually helpful for contractions per the gate-control theory.

The nurse would expect which maternal cardiovascular finding during labor? a. Increased cardiac output b. Decreased pulse rate c. Decreased white blood cell (WBC) count d. Decreased blood pressure

ANS: A During each contraction, 400 mL of blood is emptied from the uterus into the maternal vascular system. This increases cardiac output by about 51% above baseline pregnancy values at term. The heart rate increases slightly during labor. The WBC count can increase during labor. During the first stage of labor, uterine contractions cause systolic readings to increase by about 10 mm Hg. During the second stage, contractions may cause systolic pressures to increase by 30 mm Hg and diastolic readings to increase by 25 mm Hg.

Which deceleration of the fetal heart rate would not require the nurse to change the maternal position? a. Early decelerations b. Late decelerations c. Variable decelerations d. It is always a good idea to change the womans position.

ANS: A Early decelerations (and accelerations) generally do not need any nursing intervention. Late decelerations suggest that the nurse should change the maternal position (lateral); variable decelerations also require a maternal position change (side to side). Although changing positions throughout labor is recommended, it is not required in response to early decelerations.

The nurse caring for the laboring woman should understand that early decelerations are caused by: a. Altered fetal cerebral blood flow. b. Umbilical cord compression. c. Uteroplacental insufficiency. d. Spontaneous rupture of membranes.

ANS: A Early decelerations are the fetuss response to fetal head compression. Variable decelerations are associated with umbilical cord compression. Late decelerations are associated with uteroplacental insufficiency. Spontaneous rupture of membranes has no bearing on the fetal heart rate unless the umbilical cord prolapses, which would result in variable or prolonged bradycardia.

At which time is it most important to monitor for umbilical cord prolapse? a. When the fetus is crowning b. At the onset of labor c. During transitional labor d. After rupture of membranes

d. After rupture of membranes

The nurse providing care for the laboring woman comprehends that accelerations with fetal movement: a. Are reassuring. b. Are caused by umbilical cord compression. c. Warrant close observation. d. Are caused by uteroplacental insufficiency.

ANS: A Episodic accelerations in the fetal heart rate (FHR) occur during fetal movement and are indications of fetal well-being. Umbilical cord compression results in variable decelerations in the FHR. Accelerations in the FHR are an indication of fetal well-being and do not warrant close observation. Uteroplacental insufficiency would result in late decelerations in the FHR.

What does the nurse explain is used to soften the cervix with a cervical ripening agent? a. Prostaglandin gel insertion b. Intravenous oxytocin c. Warm saline douches d. Nipple stimulation

ANS: A Prostaglandin gel is inserted in the cervix and the woman remains in bed for 1 to 2 hours, being monitored for uterine contractions.

The husband of a woman in labor asks, What does it mean when the baby is at minus 1 station? After giving an explanation, what statement by the husband indicates that teaching was effective? a. Fetal head is above the ischial spines. b. Fetal head is below the ischial spines. c. Fetal head is engaged in the mothers pelvis. d. Fetal head is visible at the perineum.

ANS: A Station describes the level of the presenting part in the pelvis. It is estimated in centimeters from the level of the ischial spines. Minus stations are above the ischial spines.

A frustrated patient in labor has been affected by decreased uterine muscle tone and reports, My doctor wont induce my labor because of some silly score. He said I was a 4. What kind of magic number do I need? What is the lowest Bishop score the patient should have prior to induction? a. 6 b. 8 c. 10 d. 12

ANS: A The Bishop score evaluates the suitability of the patient for a vaginal delivery. A minimum score of 6 is recommended by the American Congress of Obstetricians and Gynecologists (ACOG).

Which factors influence cervical dilation (Select all that apply) ? (4) a. Strong uterine contractions b. The force of the presenting fetal part against the cervix c. The size of the female pelvis d. The pressure applied by the amniotic sac e. Scarring of the cervix

ANS: A, B, D, E Dilation of the cervix occurs by the drawing upward of the musculofibrous components of the cervix, which iscaused by strong uterine contractions. Pressure exerted by the amniotic fluid while the membranes are intact or by the force applied by the presenting part also can promote cervical dilation. Scarring of the cervix as a result of a previous infection or surgery may slow cervical dilation. Pelvic size does not affect cervical dilation.

What complications of overstimulation of uterine contractions may occur? (Select all that apply.) (3) a. Water intoxication b. Impaired placental exchange of oxygen and nutrients c. Increased blood pressure d. Convulsions e. Uterine rupture

ANS: A, B, E The most common complications are impaired placental exchange and uterine rupture, but water intoxication can occur due to fluid retention.

Signs that precede labor include (Select all that apply): (3) a. Lightening. b. Exhaustion. c. Bloody show. d. Rupture of membranes. e. Decreased fetal movement.

ANS: A, C, D Signs that precede labor may include lightening, urinary frequency, backache, weight loss, surge of energy, bloody show, and rupture of membranes. Many women experience a burst of energy before labor. A decrease in fetal movement is an ominous sign that does not always correlate with labor.

With regard to spinal and epidural (block) anesthesia, nurses should know that: a. This type of anesthesia is commonly used for cesarean births but is not suitable for vaginal births. b. A high incidence of after-birth headache is seen with spinal blocks. c. Epidural blocks allow the woman to move freely. d. Spinal and epidural blocks are never used together.

ANS: B Headaches may be prevented or mitigated to some degree by a number of methods. Spinal blocks may be used for vaginal births, but the woman must be assisted through labor. Epidural blocks limit the womans ability to move freely. Combined use of spinal and epidural blocks is becoming increasingly popular.

What nursing care should be provided to a woman with a third-degree laceration immediately after delivery? a. Warm compresses to the perineum b. Cold pack to the perineum c. Warm sitz bath d. Elevation of hips to prevent edema

ANS: B Ice is applied to the perineum to reduce bruising and edema.

The nurse observes the patient bearing down with contractions and crying out, The baby is coming! What is the best nursing intervention? a. Find the physician. b. Stay with the woman and use the call bell to get help. c. Send the womans partner to locate a registered nurse. d. Assist with deep breathing to slow the labor process.

ANS: B If birth appears to be imminent, the nurse should not leave the woman and should summon help with the call bell.

When caring for the laboring patient, the nurse determines that the fetus is located in the right occiput posterior (ROA). What will the nurse anticipate? a. Urinary retention b. Severe lower back pain c. A shorter labor process d. Nausea

ANS: B If the fetal occiput is in a posterior pelvic quadrant, each contraction pushes it against the mothers sacrum, resulting in persistent and poorly relieved back pain (back labor). Labor is often longer with this fetal position

What sign(s) of infection should the nurse assess for after an amniotomy? (Select all that apply.) (1) a. Oral temperature of 37 C (99.8 F) b. Increase of fetal heart rate (FHR) from 160 to 174 beats/minute c. Flecks of vernix in the amniotic fluid d. Low back pain e. Edematous labia

ANS: B Increase in the FHR above 160 beats/minute frequently precedes a womans temperature elevation. All the other options are normal findings for late pregnancy

What correctly matches the type of deceleration with its likely cause? a. Early decelerationumbilical cord compression b. Late decelerationuteroplacental inefficiency c. Variable decelerationhead compression d. Prolonged decelerationcause unknown

ANS: B Late deceleration is caused by uteroplacental inefficiency. Early deceleration is caused by head compression. Variable deceleration is caused by umbilical cord compression. Prolonged deceleration has a variety of either benign or critical causes.

Why is continuous electronic fetal monitoring usually used when oxytocin is administered? a. The mother may become hypotensive. b. Uteroplacental exchange may be compromised. c. Maternal fluid volume deficit may occur. d. Fetal chemoreceptors are stimulated.

ANS: B The uterus may contract more firmly, and the resting tone may be increased with oxytocin use. This response reduces entrance of freshly oxygenated maternal blood into the intervillous spaces, thus depleting fetal oxygen reserves. Hypotension is not a common side effect of oxytocin. All laboring women are at risk for fluid volume deficit; oxytocin administration does not increase the risk. Oxytocin affects the uterine muscles.

If an opioid antagonist is administered to a laboring woman, she should be told that: a. Her pain will decrease. b. Her pain will return. c. She will feel less anxious. d. She will no longer feel the urge to push.

ANS: B The woman should be told that the pain that was relieved by the opioid analgesic will return with administration of the opioid antagonist. Opioid antagonists, such as Narcan, promptly reverse the central nervous system (CNS) depressant effects of opioids. In addition, the antagonist counters the effect of the stress-induced levels of endorphins. An opioid antagonist is especially valuable if labor is more rapid than expected and birth is anticipated when the opioid is at its peak effect

The nurse providing care for the laboring woman realizes that variable fetal heart rate (FHR) decelerations are caused by: a. Altered fetal cerebral blood flow. b. Umbilical cord compression. c. Uteroplacental insufficiency. d. Fetal hypoxemia.

ANS: B Variable decelerations can occur any time during the uterine contracting phase and are caused by compression of the umbilical cord. Altered fetal cerebral blood flow would result in early decelerations in the FHR. Uteroplacental insufficiency would result in late decelerations in the FHR. Fetal hypoxemia would result in tachycardia initially and then bradycardia if hypoxia continues.

A multigravida at 38 weeks' gestation is admitted with painless, bright red bleeding and mild contractions every 7 to 10 minutes. Which of the following assessments should be avoided? A. Contraction monitoring B. Cervical dilation C. Maternal vital sign D. fetal heart rate

ANS: B The signs indicate placenta previa and vaginal exam to determine cervical dilation would not be done because it could cause hemorrhage. Assessing maternal vital signs can help determine maternal physiologic status. Fetal heart rate is important to assess fetal well-being and should be done. Monitoring the contractions will help evaluate the progress of labor.

With a fetus in the left-anterior breech presentation, the nurse would expect the fetal heart rate would be most audible in which of the following areas? A. In the lower-left maternal abdominal quadrant B. Above the maternal umbilicus and to the left of midline C. Above the maternal umbilicus and to the right of midline D. In the lower-right maternal abdominal quadrant

ANS: B With this presentation, the fetal upper torso and back face the left upper maternal abdominal wall. The fetal heart rate would be most audible above the maternal umbilicus and to the left of the middle. The other positions would be incorrect.

When assessing a woman in labor, the nurse is aware that the relationship of the fetal body parts to one another is called fetal: a. Lie. b. Presentation. c. Attitude. d. Position.

ANS: C Attitude is the relation of the fetal body parts to one another. Lie is the relation of the long axis (spine) of the fetus to the long axis (spine) of the mother. Presentation refers to the part of the fetus that enters the pelvic inlet first and leads through the birth canal during labor at term. Position is the relation of the presenting part to the four quadrants of the mothers pelvis.

Why is the relaxation phase between contractions important? a. The laboring woman needs to rest. b. The uterine muscles fatigue without relaxation. c. The contractions can interfere with fetal oxygenation. d. The infant progresses toward delivery at these times.

ANS: C Blood flow from the mother into the placenta gradually decreases during contractions. During the interval between contractions, the placenta refills with oxygenated blood for the fetus.

A primigravida who is 2 weeks away from her delivery date tells the obstetrical nurse that she feels like "the baby has dropped." What would be the nurse's best response to this client? A) "This feeling may be a sign that there is a complication with your pregnancy." B) "This feeling is called lightening and means that the fetus has settled into the pelvis." C) "This is a normal feeling at this stage and it is called 'Braxton-Hicks' contractions." D) "This is a normal feeling called lightening signaling that labor has begun."

B) "This feeling is called lightening and means that the fetus has settled into the pelvis."

A fetus is experiencing variable decelerations in the fetal heart rate during contractions. What is the appropriate nursing intervention for this situation? A) Have the pregnant woman walk around the room. B) Change the woman's position and give oxygen. C) Notify the healthcare practitioner immediately. D) Administer prescribed pain medications.

B) Change the woman's position and give oxygen

A nurse is caring for a client in labor. During assessment, which of the following should the nurse consider to be a common discomfort of labor? A) Exaggerated fetal movement B) Pain in the lower back C) Prolonged slowing of the fetal heart rate D) Irregular fetal heartbeat

B) Pain in the lower back

A client in the first stage of labor has an episode of bright-red bleeding. What is the best action for the nurse to take? A) Perform a vaginal examination per protocol. B) Report any bleeding at once. C) Perform an ultrasound examination, as ordered. D) Inject vitamin K, as ordered, to stop bleeding.

B) Report any bleeding at once.

A pregnant woman is having moderately strong contractions every 3 minutes lasting 50 seconds. Her cervix is dilated 6 cm. What state of labor is she experiencing? A) Stage I, latent phase B) Stage I, active phase C) Stage I, transitional phase D) Stage II

B) Stage I, active phase

The obstetrical nurse checks the station of a laboring pregnant woman and documents +5 on the patient chart. What does this number mean in terms of the delivery? A) The baby is "floating" above the mother's ischial spines. B) The fetal head is at the vaginal opening. C) The lowest part of the fetal skull is at the level of the mother's ischial spines. D) The presenting part is 5 cm above the level of the ischial spines.

B) The fetal head is at the vaginal opening.

After teaching a group of students about the maternal bony pelvis, which statement by the group indicates that the teaching was successful? A) The bony pelvis plays a lesser role during labor than soft tissue. B) The pelvic outlet is associated with the true pelvis. C) The false pelvis lies below the imaginary linea terminalis. D) The false pelvis is the passageway through which the fetus travels

B) The pelvic outlet is associated with the true pelvis.

A nurse is assessing a pregnant client whose membranes have ruptured. Which of the following findings may indicate an infection? A) Nitrazine paper remains yellow B) White or cloudy amniotic fluid C) Clear and colorless amniotic fluid D) Nitrazine paper turns blue

B) White or cloudy amniotic fluid

The nurse assisting with deliveries is aware that which of the following clients is most at risk for having a difficult delivery and possibly cesarean section. A) A woman whose baby is in a longitudinal lie B) A woman whose baby has a cephalic presentation C) A woman whose baby has engaged prior to labor D) A woman whose baby is in the footling breech position

D) A woman whose baby is in the footling breech position

The husband of a pregnant woman in labor is coaching her to perform breathing exercises to distract her from the pain. Which of the four P's of labor would the parents be? A) Passage B) Passenger C) Powers D) Psyche

D) Psyche

The nurse is caring for a laboring woman who has been given oxytocin to induce labor. She is having contractions at an interval of every 2 minutes with 90 seconds duration in between. What would be the appropriate intervention in this situation? A) Encourage the woman's coach to help her with breathing exercises. B) Provide a back rub for the woman when the pain is at its greatest intensity. C) Check her pain medication preferences and administer an analgesic, if indicated. D) Report this emergency situation to the birth attendant.

D) Report this emergency situation to the birth attendant.

A client is in the second stage of labor. The nurse should record the following information in the second stage: A) The exact time of placental delivery B) The nature of placental delivery C) The side of placental presentation D) The type of episiotomy on the client's chart

D) The type of episiotomy on the client's chart

A client is in labor, and the nurse palpates her abdomen to assess the presentation of the fetus. Which of the following presentations indicates normal labor? A) Breech presentation B) Shoulder presentation C) Brow presentation D) Vertex presentation

D) Vertex presentation

Which of the following characteristics of contractions would the nurse expect to find in a client experiencing true labor? a. Occurring at irregular intervals b. Starting mainly in the abdomen c. Gradually increasing intervals d. Increasing intensity with walking

D. With true labor, contractions increase in intensity with walking. In addition, true labor contractions occur at regular intervals, usually starting in the back and sweeping around to the abdomen. The interval of true labor contractions gradually shortens.

The nurse enters a laboring client's room. The client is complaining of intense back pain with each contraction. The nurse concludes that the fetus is likely in which of the following positions? 1. Mentum anterior. 2. Sacrum posterior. 3. Occiput posterior. 4. Scapula anterior.

3. When a fetus is in the occiput posterior position, mothers frequently complain of severe back pain. TEST-TAKING TIP: If the test taker were to view a picture of a baby in the occiput posterior position, he or she would note that the occiput of the baby lies adjacent to the coccyx of the mother. During each contraction, the occiput, therefore, is forced backward into the coccyx. This action is very painful.

While performing Leopold maneuvers on a woman in labor, the nurse palpates a hard round mass in the fundal area, a fl at surface on the left side, small objects on the right side, and a soft round mass just above the symphysis. Which of the following is a reasonable conclusion by the nurse? 1. The fetal position is transverse. 2. The fetal presentation is vertex. 3. The fetal lie is vertical. 4. The fetal attitude is fl exed.

3. With the fi ndings of a hard round mass in the fundal area and soft round mass above the symphysis, the nurse can conclude that the fetal lie is vertical.

A nurse determines that a client is carrying a fetus in the vertical lie. The nurse's judgment should be questioned if the fetal presenting part is which of the following? 1. Sacrum. 2. Occiput. 3. Mentum. 4. Scapula.

4 A fetus in a scapular presentation is in a horizontal lie. TEST-TAKING TIP: Lie is concerned with the relationship between the fetal spine and the maternal spine. When the spines are parallel, the lie is vertical (or longitudinal). When the spines are perpendicular, the lie is horizontal (or transverse). It is physiologically impossible for a baby in the horizontal lie to be delivered vaginally.

2. A woman who states that she "thinks" she is in labor enters the labor suite. Which of the following assessments will provide the nurse with the most valuable information regarding the client's labor status? 1. Leopold maneuvers. 2. Fundal contractility. 3. Fetal heart assessment. 4. Vaginal examination.

4. A vaginal examination will provide the nurse with the best information about the status of labor. TEST-TAKING TIP: Each of the assessments listed is performed on a woman who enters the labor suite for assessment. However, the only assessment that will determine whether or not a woman is in true labor is a vaginal examination. Only when there is cervical change—dilation and/or effacement—is it determined that a woman is in true labor.

A nurse is coaching a woman who is in the second stage of labor. Which of the following should the nurse encourage the woman to do? 1. Hold her breath for twenty seconds during every contraction. 2. Blow out forcefully during every contraction. 3. Push between contractions until the fetal head is visible. 4. Take a slow cleansing breath before bearing down.

4. By taking a slow, cleansing breath before pushing, the woman is waiting until the contraction builds to its peak. Her pushes will be more effective at this point in the contraction.

55. After analyzing an internal fetal monitor tracing, the nurse concludes that there is moderate short-term variability. Which of the following interpretations should the nurse make in relation to this finding? 1. The fetus is becoming hypoxic. 2. The fetus is becoming alkalotic. 3. The fetus is in the middle of a sleep cycle. 4. The fetus has a healthy nervous system.

4. Moderate variability is indicative of fetal health

A midwife advises a mother that her obstetric conjugate is of average size. How should the nurse interpret that information for the mother? 1. The anterior to posterior diameter of the pelvis will accommodate a fetus with an average-sized head. 2. The fetal head is flexed so that it is of average diameter. 3. The mother's cervix is of average dilation for the start of labor. 4. The distance between the mother's physiological retraction ring and the fetal head is of average dimensions.

1. The obstetric conjugate is the shortest anterior to posterior diameter of the pelvis. When it is of average size, it will accommodate an average-sized fetal head.

Upon examination, a nurse notes that a woman is 10 cm dilated, 100% effaced, and − 3 station. Which of the following actions should the nurse perform during the next contraction? 1. Encourage the woman to push. 2. Provide firm fundal pressure. 3. Move the client into a squat. 4. Monitor for signs of rectal pressure.

4. Monitoring for rectal pressure is appropriate at this time. TEST-TAKING TIP: Although the test taker may see in practice that women are encouraged to begin to push as soon as they become fully dilated, it is best practice to wait until the woman exhibits signs of rectal pressure. Pushing a baby who is not yet engaged may result in an overly fatigued woman or, more significantly, a prolapsed cord.

42. The nurse sees the fetal head through the vaginal introitus when a woman pushes. The nurse, interpreting this fi nding, tells the client, "You are pushing very well." In addition, the nurse could also state which of the following? 1. "The baby's head is engaged." 2. "The baby is floating." 3. "The baby is at the ischial spines." 4. "The baby is almost crowning."

4. The baby's head is almost crowning.

28. While evaluating the fetal heart monitor tracing on a client in labor, the nurse notes that there are fetal heart decelerations present. Which of the following assessments must the nurse make at this time? 1. The relationship between the decelerations and the labor contractions. 2. The maternal blood pressure. 3. The gestational age of the fetus. 4. The placement of the fetal heart electrode in relation to the fetal position.

1. The relationship between the decelerations and the contractions will determine the type of deceleration pattern. TEST-TAKING TIP: Decelerations are defi ned by their relationship to the contraction pattern. It is essential that the nurse determine which of the three types of decelerations is present. Early decelerations mirror contractions, late decelerations develop at the peak of contractions and return to baseline well after contractions are over, and variable decelerations can occur at anytime and are often unrelated to contractions.

27. While caring for a client in the transition phase of labor, the nurse notes that the fetal monitor tracing shows average short-term and long-term variability with a baseline of 142 beats per minute (bpm). What should the nurse do? 1. Provide caring labor support. 2. Administer oxygen via face mask. 3. Change the client's position. 4. Speed up the client's intravenous.

1. The tracing is showing a normal fetal heart tracing. No intervention is needed.

49. Which of the following pictures depicts a fetus in the ROP position? (SEE PICS) 1. 2. 3. 4.

1. This is a picture of a fetus in the right occiput posterior (ROP) position. 2. This is a picture of a fetus in the right occiput anterior (ROA) position. 3. This is a picture of a fetus in the left occiput posterior (LOP) position. 4. This is a picture of a fetus in the left occiput anterior (LOA) position.

25. The nurse auscultates a fetal heart rate of 152 on a client in early labor. Which of the following actions by the nurse is appropriate? 1. Inform the mother that the rate is normal. 2. Reassess in 5 minutes to verify the results. 3. Immediately report the rate to the healthcare practitioner. 4. Place the client on her left side and apply oxygen by face mask.

1. This is the correct response. A fetal heart rate of 152 is normal.

Which nursing assessment indicates that a woman who is in second-stage labor is almost ready to give birth? a. The fetal head is felt at 0 station during vaginal examination. b. Bloody mucus discharge increases. c. The vulva bulges and encircles the fetal head. d. The membranes rupture during a contraction.

ANS: C During the active pushing (descent) phase, the woman has strong urges to bear down as the presenting part of the fetus descends and presses on the stretch receptors of the pelvic floor. The vulva stretches and begins to bulge encircling the fetal head. Birth of the head occurs when the station is +4. A 0 station indicates engagement. Bloody show occurs throughout the labor process and is not an indication of an imminent birth. Rupture of membranes can occur at any time during the labor process and does not indicate an imminent birth.

The nurse caring for a woman in labor understands that prolonged decelerations: a. Are a continuing pattern of benign decelerations that do not require intervention. b. Constitute a baseline change when they last longer than 5 minutes. c. Usually are isolated events that end spontaneously. d. Require the usual fetal monitoring by the nurse.

ANS: C Prolonged decelerations usually are isolated events that end spontaneously. However, in certain combinations with late and/or variable decelerations, they are a danger sign that requires the nurse to notify the physician or midwife immediately. A deceleration that lasts longer than 10 minutes constitutes a baseline change.

While providing care to a patient in active labor, the nurse should instruct the woman that: a. The supine position commonly used in the United States increases blood flow. b. The all fours position, on her hands and knees, is hard on her back. c. Frequent changes in position will help relieve her fatigue and increase her comfort. d. In a sitting or squatting position, her abdominal muscles will have to work harder.

ANS: C Frequent position changes relieve fatigue, increase comfort, and improve circulation. Blood flow can be compromised in the supine position; any upright position benefits cardiac output. The all fours position is used to relieve backache in certain situations. In a sitting or squatting position, the abdominal muscles work in greater harmony with uterine contractions.

Nurses with an understanding of cultural differences regarding likely reactions to pain may be better able to help clients. Nurses should know that _____ women may be stoic until late in labor, when they may become vocal and request pain relief. a. Chinese b. Arab or Middle Eastern c. Hispanic d. African-American

ANS: C Hispanic women may be stoic early and more vocal and ready for medications later. Chinese women may not show reactions to pain. Medical interventions must be offered more than once. Arab or Middle Eastern women may be vocal in response to labor pain from the start. They may prefer pain medications. African-American women may express pain openly; use of medications for pain is more likely to vary with the individual.

The laboring woman who imagines her body opening to let the baby out is using a mental technique called: a. Dissociation. b. Effleurage. c. Imagery. d. Distraction.

ANS: C Imagery is a technique of visualizing images that will assist the woman in coping with labor. Dissociation helps the woman learn to relax all muscles except those that are working. Effleurage is self-massage. Distraction can be used in the early latent phase by having the woman engage in another activity.

What is the most important nursing intervention during the fourth stage of labor? a. Monitor the frequency and intensity of contractions. b. Provide comfort measures. c. Assess for hemorrhage. d. Promote bonding.

ANS: C Immediately after giving birth, every woman is assessed for signs of hemorrhage

The nurse caring for the woman in labor should understand that maternal hypotension can result in: a. Early decelerations. b. Fetal dysrhythmias. c. Uteroplacental insufficiency. d. Spontaneous rupture of membranes.

ANS: C Low maternal blood pressure reduces placental blood flow during uterine contractions and results in fetal hypoxemia. Maternal hypotension is not associated with early decelerations, fetal dysrhythmias, or spontaneous rupture of membranes.

Which narcotic antagonist is used to reverse narcotic-induced respiratory depression? a. Hydroxyzine (Vistaril) b. Phenobarbital c. Naloxone (Narcan) d. Nitrous oxide

ANS: C Naloxone (Narcan) is used to reverse respiratory depression caused by narcotics.

An infant is delivered with the use of forceps. What should the nurse assess for in the newborn? a. Loss of hair from contact with forceps b. Sacral hematoma c. Facial asymmetry d. Shoulder dislocation

ANS: C Pressure from forceps may injure the infants facial nerve, which is evidenced by facial asymmetry.

While caring for a laboring woman, the nurse notices a pattern of variable decelerations in fetal heart rate with uterine contractions. What is the nurses initial action? a. Stop the oxytocin infusion. b. Increase the intravenous flow rate. c. Reposition the woman on her side. d. Start oxygen via nasal cannula

ANS: C Repositioning the woman is the first response to a pattern of variable decelerations. If the decelerations continue, then oxygen should be administered and/or the flow rate of oxygen should be increased.

A woman has decided to hire a doula to work with her during labor and delivery. Which of the following actions would be appropriate for the nurse to delegate to the doula? Select all that apply. (2) 1. Give the woman a back rub. 2. Assist the woman with her breathing. 3. Assess the fetal heart rate. 4. Check the woman's blood pressure. 5. Regulate the woman's intravenous infusion rate.

1 and 2 are correct. 1. An appropriate action by the doula is giving the woman a back massage. 2. An appropriate action by the doula is to assist the laboring woman with her breathing.

A client is in the third stage of labor. Which of the following assessments should the nurse make/observe for? Select all that apply. (2) 1. Lengthening of the umbilical cord. 2. Fetal heart assessment after each contraction. 3. Uterus rising in the abdomen and feeling globular. 4. Rapid cervical dilation to ten centimeters. 5. Maternal complaints of intense rectal pressure.

1 and 3 are correct. 1. This is a sign of placental separation. 3. This is a sign of placental separation.

1. A client enters the labor and delivery suite stating that she thinks she is in labor. Which of the following information about the woman should the nurse note from the woman's prenatal record before proceeding with the physical assessment? Select all that apply. (4) 1. Weight gain. 2. Ethnicity and religion. 3. Age. 4. Type of insurance. 5. Gravidity and parity.

1, 2, 3, and 5 are correct. 1. Before proceeding with a physical assessment, the nurse should check the client's weight gain reported in her prenatal record. 2. The client's ethnicity and religion should be noted before physical assessment. This allows the nurse to proceed in a culturally sensitive manner. 3. The client's age should also be noted before the physical assessment is begun. 5. The client's gravidity and parity—how many times she has been pregnant and how many times she has given birth—should also be noted before a physical assessment is begun

When during the latent phase of labor should the nurse assess the fetal heart pattern of a low-risk woman, G1 P0000? Select all that apply. (4) 1. After vaginal examinations. 2. Before administration of analgesics. 3. Periodically at the end of a contraction. 4. Every ten minutes. 5. Before ambulating.

1, 2, 3, and 5 are correct. 1. The nurse should assess the fetal heart after all vaginal examinations. 2. The nurse should assess the fetal heart before giving the mother any analgesics. 3. The fetal heart should be assessed periodically at the end of a contraction. 5. The nurse should assess the fetal heart before the woman ambulates.

4. A woman has just arrived at the labor and delivery suite. To report the client's status to her primary healthcare practitioner, which of the following assessments should the nurse perform? Select all that apply. (3) 1. Fetal heart rate. 2. Contraction pattern. 3. Urinalysis. 4. Vital signs. 5. Biophysical profile.

1, 2, and 4 are correct. 1. The nurse should assess the fetal heart before reporting the client's status to the healthcare provider. 2. The nurse should assess the contraction pattern before reporting the client's status. 4. The nurse should assess the woman's vital signs before reporting her status.

The nurse is interpreting the fetal monitor tracing below. Which of the following actions should the nurse take at this time? (SEE PIC) 1. Provide caring labor support. 2. Administer oxygen via tight-fitting face mask. 3. Turn the woman on her side. 4. Apply the oxygen saturation electrode to the mother.

1. Because the variability is moderate (6 to 25 bpm wide), the nurse can conclude that the baby is well and that caring labor support is indicated.

Which of the following actions would the nurse expect to perform immediately before a woman is to have regional anesthesia? Select all that apply. (3) 1. Assess fetal heart rate. 2. Infuse 1,000 mL of Ringer's lactate. 3. Place the woman in the Trendelenburg position. 4. Monitor blood pressure every 5 minutes for 15 minutes. 5. Have the woman empty her bladder.

1. Before a woman is given regional anesthesia, the nurse should assess the fetal heart rate. 2. The nurse should receive an order to infuse Ringer's lactate before the woman is given regional anesthesia. 5. The nurse should ask the woman to empty her bladder.

On examination of a full-term primipara, a labor nurse notes: active labor, right occipitoanterior (ROA), 7 cm dilated, and +3 station. Which of the following should the nurse report to the physician? 1. Descent is progressing well. 2. Fetal head is not yet engaged. 3. Vaginal delivery is imminent. 4. External rotation is complete.

1. Descent is progressing well. The presenting part is 3 centimeters below the ischial spines. TEST-TAKING TIP: This question includes a number of concepts. Descent and station are discussed in answer options 1 and 2. The dilation of the cervix, which is related to the fact that the woman is a primigravida, is discussed in choice 3. And one of the cardinal moves of labor— external rotation—is included in choice 4. The test taker must be prepared to answer questions that are complex and that include diverse information. In a 7 cm dilated primipara with a baby at +3 station, vaginal delivery is not imminent, but the fetal head is well past engagement and descent is progressing well. External rotation has not yet occurred because the baby's head has not yet been birthed.

A nurse concludes that a woman is in the latent phase of labor. Which of the following signs/symptoms would lead a nurse to that conclusion? 1. The woman talks and laughs during contractions. 2. The woman complains about severe back labor. 3. The woman performs effleurage during a contraction. 4. The woman asks to go to the bathroom to defecate.

1. Talking and laughing are characteristic behaviors of the latent phase.

When would the nurse expect to see the fetal heart changes noted on the fetal monitor tracing shown below? 1. During fetal movement. 2. After the administration of analgesics. 3. When the fetus is acidotic. 4. With poor placental perfusion.

1. The fetal heart rate normally accelerates during fetal movement. TEST-TAKING TIP: Fetal heart accelerations, defined as a rise in the fetal heart rate above baseline of at least 15 bpm and that lasts a minimum of 15 seconds, are usually a sign of fetal well-being. When the baby is healthy, they are almost always noted during periods of fetal movement. Similar to what occurs in a runner, with increased movement, the fetal heart rate speeds up to accommodate increasing energy needs.

A client in labor is talkative and happy. How many centimeters dilated would a maternity nurse suspect that the client is at this time? 1. 2 cm. 2. 4 cm. 3. 8 cm. 4. 10 cm.

1. The nurse would expect the woman to be 2 cm dilated. 2. At 4 cm, the woman is entering the active phase of labor. 3. At 8 cm, the woman is in the transition phase of labor. 4. At 10 cm, the woman is in the second stage of labor.

The labor and delivery nurse performs Leopold maneuvers. A soft round mass is felt in the fundal region. A fl at object is noted on the left and small objects are noted on the right of the uterus. A hard round mass is noted above the symphysis. Which of the following positions is consistent with these findings? 1. Left occipital anterior (LOA). 2. Left sacral posterior (LSP). 3. Right mentum anterior (RMA). 4. Right sacral posterior (RSP).

1. The nurse's findings upon performing Leopold maneuvers indicate that the fetus is in the left occiput anterior (LOA) position—that is, the fetal back is felt on the mother's left side, the small parts are felt on her right side, the buttocks are felt in the fundal region, and the head is felt above her symphysis.

The nurse is assessing an internal fetal heart monitor tracing of an unmedicated, full-term gravida who is in transition. Which of the following heart rate patterns would the nurse interpret as normal? 1. Baseline of 140 to 150 with V-shaped decelerations to 120 unrelated to contractions. 2. Baseline of 140 to 150 with decelerations to 100 that mirror each of the contractions. 3. Baseline of 140 to 142 with decelerations to 120 that return to baseline after the end of the contractions. 4. Baseline of 140 to 142 with no obvious decelerations or accelerations.

2. A baseline FHR of 140 to 150 is a baseline showing moderate, or normal, variability. Decelerations that mirror contractions are defined as early decelerations. These are related to head compression and are expected during transition and second stage labor. TEST-TAKING TIP: The test taker must be prepared to differentiate between normal situations and obstetric emergencies. Even though there are decelerations in choice 2, they are early decelerations and they are expected because the woman is currently in the transition phase of the first stage of labor.

22. In addition to breathing with contractions, the nurse should encourage women in the first stage of labor to perform which of the following therapeutic actions? 1. Lying in the lithotomy position. 2. Performing effleurage. 3. Practicing Kegel exercises. 4. Pushing with each contraction.

2. Effleurage is a light massage that can soothe the mother during labor. TEST-TAKING TIP: There are a number of actions that mothers can take that can support their breathing during labor. Walking, swaying, and rocking can all help a woman during the process. Effl eurage, the light massaging of the abdomen or thighs, is often soothing for laboring mothers.

Immediately following administration of an epidural anesthesia, the nurse must monitor the mother for which of the following side effects? 1. Paresthesias in her feet and legs. 2. Drop in blood pressure. 3. Increase in central venous pressure. 4. Fetal heart accelerations.

2. Hypotension is a very common side effect of regional anesthesia

34. The nurse is assessing a client who states, "I think I'm in labor." Which of the following findings would positively confirm the client's belief? 1. She is contracting q 5 min × 60 sec. 2. Her cervix has dilated from 2 to 4 cm. 3. Her membranes have ruptured. 4. The fetal head is engaged.

2. Once the cervix begins to dilate, a client is in true labor. 1. Women may contract without being in true labor. 3. Membranes can rupture before true labor begins. 4. Engagement can occur before true labor begins.

The nurse is assessing the fetal station during a vaginal examination. Which of the following structures should the nurse palpate? 1. Sacral promontory. 2. Ischial spines. 3. Cervix. 4. Symphysis pubis.

2. Station is assessed by palpating the ischial spines.

During a vaginal examination, the nurse palpates fetal buttocks that are facing the left posterior and are 1 cm above the ischial spines. Which of the following is consistent with this assessment? 1. LOA − 1 station. 2. LSP − 1 station. 3. LMP +1 station. 4. LSA +1 station.

2. The LSP position is the correct answer. The fetal buttocks (S or sacrum) are facing toward the mother's left posterior (LP) and buttocks at − 1 station are 1 cm above the ischial spines. TEST-TAKING TIP: If the test taker understands the defi nition of station, he or she could easily eliminate two of the four responses in this question. When the presenting part of the fetus is at zero (0) station, the part is at the same level as an imaginary line between the mother's ischial spines. When the presenting part is above the spines, the station is negative (-). When the presenting part has moved past the spines, the station is defi ned as positive (+). Because the question states that the nurse palpated the buttocks above the spines, the station is negative. This effectively eliminates the two answer options that include a positive station.

A nurse is assessing the vital signs of a client in labor at the peak of a contraction. Which of the following findings would the nurse expect to see? 1. Decreased pulse rate. 2. Hypertension. 3. Hyperthermia. 4. Decreased respiratory rate.

2. The blood pressure rises dramatically.

The practitioner is performing a fetal scalp stimulation test. Which of the following fetal responses would the nurse expect to see? 1. Spontaneous fetal movement. 2. Fetal heart acceleration. 3. Increase in fetal heart variability. 4. Resolution of late decelerations.

2. The fetal heart should accelerate in response to scalp stimulation. TEST-TAKING TIP: The fetal scalp stimulation test is performed by the healthcare practitioner when the fetal heart pattern is equivocal. For example, if the variability is questionable, the practitioner may perform the stimulation test. If the fetal heart rate accelerates in response to the test, the nurse interprets the response as a positive sign.

A woman, G1 P0000, 40 weeks' gestation, entered the labor suite stating that she is in labor. Upon examination it is noted that the woman is 2 cm dilated, 30% effaced, contracting every 12 min × 30 sec. Fetal heart rate is in the 140s with good variability and spontaneous accelerations. What should the nurse conclude when reporting the findings to the primary healthcare practitioner? 1. The woman is at high risk and should be placed on tocolytics. 2. The woman is in early labor and could be sent home. 3. The woman is at high risk and could be induced. 4. The woman is in active labor and should be admitted to the unit.

2. The woman is in early labor. There is no need for her to be hospitalized at this time. TEST-TAKING TIP: The key facts that the test taker should attend to in this question about a primigravida are the cervical dilation, the contraction pattern, and the fetal heart pattern. The woman is clearly in the latent phase because she is only 2 cm dilated, is 30% effaced, and is contracting infrequently q 12 minutes with short duration. Plus, the fetal heart rate is excellent. She could be sent home to labor in comfort.

A client in labor, G2 P1001, was admitted 1 hour ago at 2 cm dilated and 50% effaced. She was talkative and excited at that time. During the past 10 minutes she has become serious, closing her eyes and breathing rapidly with each contraction. Which of the following is an accurate nursing assessment of the situation? 1. The client had poor childbirth education prior to labor. 2. The client is exhibiting an expected behavior for labor. 3. The client is becoming hypoxic and hypercapnic. 4. The client needs her alpha-fetoprotein levels checked.

2. The woman is showing expected signs of the active phase of labor. TEST-TAKING TIP: The test taker must be familiar with the different phases of the fi rst stage of labor: latent, active, and transition. The multiparous woman in the scenario entered the labor suite in the latent phase of labor when being talkative and excited is normal, but after 1 hour she has progressed into the active phase of labor in which being serious and breathing rapidly with contractions are expected behaviors.

23. A client is in the second stage of labor. She falls asleep immediately after a contraction. Which of the following actions should the nurse perform at this time? 1. Awaken the woman and remind her to push. 2. Cover the woman's perineum with a sheet. 3. Assess the woman's blood pressure and pulse. 4. Administer oxygen to the woman via face mask.

2. The woman's privacy should be maintained while she is resting. TEST-TAKING TIP: Because the woman is in second stage, she is pushing with contractions. If she is very tired, she is likely to fall asleep immediately following a contraction. It is important for the nurse to maintain the woman's privacy bycovering her perineum with a sheet between contractions. It would also be appropriate to awaken the woman at the beginning of the next contraction

A woman had a baby by normal spontaneous delivery 10 minutes ago. The nurse notes that a gush of blood was just expelled from the vagina and the umbilical cord lengthened. What should the nurse conclude? 1. The woman has an internal laceration. 2. The woman is about to deliver the placenta. 3. The woman has an atonic uterus. 4. The woman is ready to expel the cord bloods.

2. These are signs of placental delivery.

A client who is 7 cm dilated and 100% effaced is breathing at a rate of 50 breaths per minute during contractions. Immediately after a contraction, she complains of tingling in her fi ngers and some light-headedness. Which of the following actions should the nurse take at this time? 1. Assess the blood pressure. 2. Have the woman breathe into a bag. 3. Turn the woman onto her side. 4. Check the fetal heart rate.

2. This client is showing signs of hyperventilation. The symptoms will likely subside if she rebreathes her exhalations. TEST-TAKING TIP: It is essential that the test taker attend to the clues in the question and not assume that other issues may be occurring. This client is lightheaded as a result of being tachypneic during contractions. Hyperventilation, which can result from tachypnea, is characterized by tingling and lightheadedness. Rebreathing her air should rectify the problem.

A woman is in the transition phase of labor. Which of the following comments should the nurse expect to hear? 1. "I am so excited to be in labor." 2. "I can't stand this pain any longer!" 3. "I need ice chips because I'm so hot." 4. "I have to push the baby out right now!"

2. This comment is consistent with a woman in the transition phase of stage 1.

A woman is in active labor and is being monitored electronically. She has just received Stadol 2 mg IM for pain. Which of the following fetal heart responses would the nurse expect to see on the internal monitor tracing? 1. Variable decelerations. 2. Late decelerations. 3. Decreased variability. 4. Transient accelerations.

3. Analgesics are central nervous system (CNS) depressants. The variability of the fetal heart rate, therefore, will be decreased.

A primigravida is pushing with contractions. The nurse notes that the woman's perineum is beginning to bulge and that there is an increase in bloody show. Which of the following actions by the nurse is appropriate at this time? 1. Report the findings to the woman's healthcare practitioner. 2. Immediately assess the woman's pulse and blood pressure. 3. Continue to provide encouragement during each contraction. 4. Place the client on her side with oxygen via face mask.

3. Because this is a normal finding, the nurse should continue to provide labor support and encouragement. TEST-TAKING TIP: The bulging perineum is an indication that the baby is descending in the birth canal and the bloody show results from injury to the capillaries in the mother's cervix. Because this woman is a primigravida, she will likely need to push for many more minutes so it is not necessary to notify the healthcare provider until additional signs are noted

A multipara, LOA, station +3, who has had no pain medication during her labor, is now in stage 2. She states that her pain is 6 on a 10-point scale and that she wants an epidural. Which of the following responses by the nurse is appropriate? 1. "Epidurals do not work well when the pain level is above level 5." 2. "I will contact the doctor to get an order for an epidural right away." 3. "The baby is going to be born very soon. It is really too late for an epidural." 4. "I will check the fetal heart rate. You can have an epidural if it is over 120."

3. Because this woman is a multipara, the position is LOA, and the station is +3, this is an accurate statement. TEST-TAKING TIP: The average length of the second stage of labor for multiparas is about 15 minutes, whereas the average time for an epidural to be inserted and to take effect is approximately 20 minutes. In addition, the fetus in the scenario has already descended to +3 station and is in the optimal position for delivery—LOA. It is very likely that this baby will be born in a few contractions. The nurse should encourage the client to continue pushing with her contractions.

56. When would the nurse expect to see the fetal monitor tracing shown below? (SEE PIC) 1. During latent phase of labor. 2. During an epidural insertion. 3. During second stage of labor. 4. During delivery of the placenta.

3. Early decelerations are frequently seen during the second stage of labor

On vaginal examination, it is noted that a woman with a well-functioning epidural is in the second stage of labor. The station is -2 and the baseline fetal heart rate is 130 with no decelerations. Which of the following nursing actions is appropriate at this time? 1. Coach the woman to hold her breath while pushing 3 to 4 times with each contraction. 2. Administer oxygen via face mask at 8 to 10 liters per minute. 3. Delay pushing until the baby descends further and the mother has a strong urge to push. 4. Place the woman on her side and assess her oxygen saturation.

3. Once the woman has a strong urge to push, then she should be encouraged to push against an open glottis to birth the baby.

The nurse knows that which of the following responses is the primary rationale for the inclusion of the information taught in childbirth education classes? 1. Mothers who are performing breathing exercises during labor refrain from yelling. 2. Breathing and relaxation exercises are less exhausting than crying and moaning. 3. Knowledge learned at childbirth education classes helps to break the fear-tension pain cycle. 4. Childbirth education classes help to promote positive maternal-newborn bonding.

3. Some of the techniques learned at childbirth education classes are meant to break the fear-tension-pain cycle.

When performing Leopold maneuvers, the nurse notes that the fetus is in the left occiput anterior position. Which is the best position for the nurse to place a fetoscope to hear the fetal heartbeat? 1. Left upper quadrant. 2. Right upper quadrant. 3. Left lower quadrant. 4. Right lower quadrant.

3. The fetoscope should be placed in the left lower quadrant for a fetus positioned in the LOA position as described in the question. TEST-TAKING TIP: The fetal heart is best heard through the fetal back. Because, as determined by doing Leopold maneuvers, the baby is LOA, the fetal back (and, hence, the fetal heart) is in the left lower quadrant.

A gravid client at term called the labor suite at 7:00 p.m. questioning whether she was in labor. The nurse determined that the client was likely in labor after the client stated: 1. "At 5:00 p.m., the contractions were about 5 minutes apart. Now they're about 7 minutes apart." 2. "I took a walk at 5:00 p.m., and now I talk through my contractions easier than I could then." 3. "I took a shower about a half hour ago. The contractions hurt more than they did before." 4. "I had some tightening in my belly late this afternoon, and I still feel it after waking up from my nap."

3. This response indicates that the labor contractions are increasing in intensity TEST-TAKING TIP: The test taker should review the labor contraction definitions of frequency, duration, and intensity. As labor progresses, the frequency of contractions decreases but the duration and the intensity, or the length and strength, of the contractions increase. The nurse notes the change in intensity when he or she palpates the fundus of the uterus, and the client subjectively complains of increasing pain.

One hour ago, a multipara was examined with the following results: 8 cm, 50% effaced, and +1 station. She is now pushing with contractions and the fetal head is seen at the vaginal introitus. The nurse concludes that the client is now: 1. 9 cm dilated, 70% effaced, and +2 station. 2. 9 cm dilated, 80% effaced, and +3 station. 3. 10 cm dilated, 90% effaced, and +4 station. 4. 10 cm dilated, 100% effaced, and +5 station.

4. The cervix is fully dilated and fully effaced and the baby is low enough to be seen through the vaginal introitus. TEST-TAKING TIP: To answer this question, the test taker must methodically evaluate each of the given responses. Once the nurse determines that a woman is not yet fully dilated or effaced, it can be determined that the woman is still in stage 1 of labor. Choice 3 does show a woman who is fully dilated but who is yet to efface fully and whose baby is still above the vaginal introitus. Only choice 4 meets all criteria set forth in the question.

A woman, 40 weeks' gestation, calls the labor unit to see whether or not she should go to the hospital to be evaluated. Which of the following statements by the woman indicates that she is probably in labor and should proceed to the hospital? 1. "The contractions are 5 to 20 minutes apart." 2. "I saw a pink discharge on the toilet tissue when I went to the bathroom." 3. "I have had cramping for the past 3 or 4 hours." 4. "The contractions are about a minute long and I am unable to talk through them."

4. This client is exhibiting clear signs of true labor. Not only are the contractions lasting a full minute but she is stating that they are so uncomfortable that she is unable to speak through them. She should be seen.

Which of the following pictures depicts a fetus in the LSA position? (see pics) 1. 2. 3. 4.

4. This is a picture of a fetus in the left sacral anterior (LSA) position. 1. This is a picture of a fetus in the right sacral posterior (RSP) position. 2. This is a picture of a fetus in the right sacral anterior (RSA) position. 3. This is a picture of a fetus in the left sacral posterior (LSP) position. TEST-TAKING TIP: The position of a fetus is defined as the direction in which the fetal presenting part is pointed in relation to the maternal pelvis, that is, the mother's left/right and anterior/posterior. In other words, the test taker must determine toward which part of the mother the presenting part of the fetus is pointing. In this question, for example, the sacrum of the fetus is pointing toward the left anterior of the mother.

47. A woman who is in active labor is told by her obstetrician, "Your baby is in the flexed attitude." When she asks the nurse what that means, what should the nurse say? 1. The baby is in the breech position. 2. The baby is in the horizontal lie. 3. The baby's presenting part is engaged. 4. The baby's chin is resting on its chest.

4. When the baby's chin is on his or her chest, the baby is in the flexed attitude.

Following delivery, a client feels the urge to void, but is unable to do so after an extended period of time. What immediate action should the nurse take? A) Report it to the practitioner. B) Catheterize the bladder. C) Apply suprapubic pressure. D) Reassure the client.

A) Report it to the practitioner.

The nurse practitioner examining a pregnant woman in labor notes that the fetal spine is parallel to the woman's spine. What is the term for this relationship of the fetal body to the maternal body? A) Lie B) Presentation C) Station D) Engagement

A) Lie

A nurse is performing fundal massage for a client. What precaution should the nurse take when giving a fundal massage? A) Never massage a contracted fundus. B) Avoid placing a hand over the symphysis pubis. C) Report cramps immediately to physician. D) Avoid applying heat to relieve cramps.

A) Never massage a contracted fundus.

Fetal well-being during labor is assessed by: a. The response of the fetal heart rate (FHR) to uterine contractions (UCs). b. Maternal pain control. c. Accelerations in the FHR. d. An FHR above 110 beats/min.

ANS: A Fetal well-being during labor can be measured by the response of the FHR to UCs. In general, reassuring FHR patterns are characterized by an FHR baseline in the range of 110 to 160 beats/min with no periodic changes, a moderate baseline variability, and accelerations with fetal movement. Maternal pain control is not the measure used to determine fetal well-being in labor. Although FHR accelerations are a reassuring pattern, they are only one component of the criteria by which fetal well-being is assessed. Although an FHR above 110 beats/min may be reassuring, it is only one component of the criteria by which fetal well-being is assessed. More information would be needed to determine fetal well-being.

Which description of the four stages of labor is correct for both definition and duration? a. First stage: onset of regular uterine contractions to full dilation; less than 1 hour to 20 hours b. Second stage: full effacement to 4 to 5 cm; visible presenting part; 1 to 2 hours c. Third state: active pushing to birth; 20 minutes (multiparous women), 50 minutes (first-timer) d. Fourth stage: delivery of the placenta to recovery; 30 minutes to 1 hour

ANS: A Full dilation may occur in less than 1 hour, but in first-time pregnancies it can take up to 20 hours. The second stage extends from full dilation to birth and takes an average of 20 to 50 minutes, although 2 hours is still considered normal. The third stage extends from birth to expulsion of the placenta and usually takes a few minutes. The fourth stage begins after expulsion of the placenta and lasts until homeostasis is reestablished (about 2 hours).

What does meconium-stained amniotic fluid indicate when the infant is in a vertex presentation? a. Fetal distress b. Fetal maturity c. Intact gastrointestinal tract d. Dehydration in the mother

ANS: A Green-stained amniotic fluid means that the fetus passed the first stool before birth, and it is an indicator of fetal compromise.

A woman is 7 cm dilated, and her contractions are 3 minutes apart. When she begins cursing at her birthing coach and the nurse, what does the nurse assess as the most likely explanation for the womans change in behavior? a. Labor has progressed to the transition phase. b. She lacked adequate preparation for the labor experience. c. The woman would benefit from a different form of analgesia. d. The contractions have increased from mild to moderate intensity.

ANS: A If a woman suddenly loses control and becomes irritable, suspect that she has progressed to the transition stage of labor.

Which presentation is described accurately in terms of both presenting part and frequency of occurrence? a. Cephalic: occiput; at least 95% b. Breech: sacrum; 10% to 15% c. Shoulder: scapula; 10% to 15% d. Cephalic: cranial; 80% to 85%

ANS: A In cephalic presentations (head first), the presenting part is the occiput; this occurs in 96% of births. In a breech birth, the sacrum emerges first; this occurs in about 3% of births. In shoulder presentations, the scapula emerges first; this occurs in only 1% of births.

Vaginal examination reveals the presenting part is the infants head, which is well flexed on the chest. What is this presentation? a. Vertex b. Military c. Brow d. Face

ANS: A In the vertex presentation, the fetal head is the presenting part. The head is fully flexed on the chest.

While evaluating an external monitor tracing of a woman in active labor, the nurse notes that the fetal heart rate (FHR) for five sequential contractions begins to decelerate late in the contraction, with the nadir of the decelerations occurring after the peak of the contraction. The nurses first priority is to: a. Change the womans position. b. Notify the care provider. c. Assist with amnioinfusion. d. Insert a scalp electrode.

ANS: A Late decelerations may be caused by maternal supine hypotension syndrome. They usually are corrected when the woman turns on her side to displace the weight of the gravid uterus from the vena cava. If the fetus does not respond to primary nursing interventions for late decelerations, the nurse would continue with subsequent intrauterine resuscitation measures, including notifying the care provider. An amnioinfusion may be used to relieve pressure on an umbilical cord that has not prolapsed. The FHR pattern associated with this situation most likely reveals variable deceleration. A fetal scalp electrode would provide accurate data for evaluating the well-being of the fetus; however, this is not a nursing intervention that would alleviate late decelerations, nor is it the nurses first priority

A woman in active labor receives an analgesic opioid agonist. Which medication relieves severe, persistent, or recurrent pain; creates a sense of well-being; overcomes inhibitory factors; and may even relax the cervix but should be used cautiously in women with cardiac disease? a. Meperidine (Demerol) b. Promethazine (Phenergan) c. Butorphanol tartrate (Stadol) d. Nalbuphine (Nubain)

ANS: A Meperidine is the most commonly used opioid agonist analgesic for women in labor throughout the world. It overcomes inhibitory factors in labor and may even relax the cervix. Because tachycardia is a possible adverse reaction, meperidine is used cautiously in women with cardiac disease. Phenergan is an ataractic (tranquilizer) that may be used to augment the desirable effects of the opioid analgesics but has few of the undesirable effects of those drugs. Stadol and Nubain are opioid agonist-antagonist analgesics.

Perinatal nurses are legally responsible for: a. Correctly interpreting fetal heart rate (FHR) patterns, initiating appropriate nursing interventions, and documenting the outcomes. b. Greeting the client on arrival, assessing her, and starting an intravenous line. c. Applying the external fetal monitor and notifying the care provider. d. Making sure that the woman is comfortable.

ANS: A Nurses who care for women during childbirth are legally responsible for correctly interpreting FHR patterns, initiating appropriate nursing interventions based on those patterns, and documenting the outcomes of those interventions. Greeting the client, assessing her, and starting an IV; applying the external fetal monitor and notifying the care provider; and making sure the woman is comfortable may be activities that a nurse performs, but they are not activities for which the nurse is legally responsible.

A woman 2 weeks past her expected delivery date is receiving an oxytocin infusion to induce labor and begins to have contractions every 90 seconds. What is the nurses initial action? a. Stop the oxytocin infusion. b. Continue the infusion and report the findings to the physician. c. Turn her on her left side and reassess the contractions. d. Administer oxygen by mask.

ANS: A Oxytocin is discontinued if signs of fetal compromise or excessive uterine contractions occur.

To care for a laboring woman adequately, the nurse understands that the __________ stage of labor varies the most in length? a. First b. Second c. Third d. Fourth

ANS: A The first stage of labor is considered to last from the onset of regular uterine contractions to full dilation of the cervix. The first stage is much longer than the second and third stages combined. In a first-time pregnancy the first stage of labor can take up to 20 hours. The second stage of labor lasts from the time the cervix is fully dilated to the birth of the fetus. The average length is 20 minutes for a multiparous woman and 50 minutes for a nulliparous woman. The third stage of labor lasts from the birth of the fetus until the placenta is delivered. This stage may be as short as 3 minutes or as long as 1 hour. The fourth stage of labor, recovery, lasts about 2 hours after delivery of the placenta.

A pregnant woman asks the nurse, Will I be able to have a vaginal delivery? The nurse knows that which is the most favorable pelvic type for vaginal birth? a. Gynecoid b. Android c. Anthropoid d. Platypelloid

ANS: A The gynecoid pelvis is the typical female pelvis and is most favorable for vaginal birth.

An 18-year-old primigravida is 4 cm dilated and her contractions are 5 minutes apart. She received little prenatal care and had no childbirth preparation. She is crying loudly and shouting, Please give me something for the pain. I cant take the pain! What is the priority nursing diagnosis? a. Pain related to uterine contractions b. Knowledge deficit related to the birth experience c. Ineffective coping related to inadequate preparation for labor d. Risk for injury related to lack of prenatal care

ANS: A The most important issue for this woman, at this time, is effective pain management

In relation to primary and secondary powers, the maternity nurse comprehends that: a. Primary powers are responsible for effacement and dilation of the cervix. b. Effacement generally is well ahead of dilation in women giving birth for the first time; they are closer together in subsequent pregnancies. c. Scarring of the cervix caused by a previous infection or surgery may make the delivery a bit more painful, but it should not slow or inhibit dilation. d. Pushing in the second stage of labor is more effective if the woman can breathe deeply and control some of her involuntary needs to push, as the nurse directs.

ANS: A The primary powers are responsible for dilation and effacement; secondary powers are concerned with expulsion of the fetus. Effacement generally is well ahead of dilation in first-timers; they are closer together in subsequent pregnancies. Scarring of the cervix may slow dilation. Pushing is more effective and less fatiguing when the woman begins to push only after she has the urge to do so.

A woman requests a pudendal block to manage her labor pain. What statement by the woman indicates a need for further explanation about the pudendal block? a. I'm having a contraction. Can I get the pudendal block now? b. Ill get the pudendal block right before I deliver. c. The nurse midwife will insert the needles into my vagina. d. It takes a few minutes after the medicine is administered to make me feel numb.

ANS: A The pudendal block does not block pain from contractions and is given just before birth.

The nurse knows that proper placement of the tocotransducer for electronic fetal monitoring is located: a. Over the uterine fundus. b. On the fetal scalp. c. Inside the uterus. d. Over the mothers lower abdomen.

ANS: A The tocotransducer monitors uterine activity and should be placed over the fundus, where the most intensive uterine contractions occur. The tocotransducer is for external use.

In assisting with the two factors that have an effect on fetal status (i.e., pushing and positioning), nurses should: a. Encourage the womans cooperation in avoiding the supine position. b. Advise the woman to avoid the semi-Fowler position. c. Encourage the woman to hold her breath and tighten her abdominal muscles to produce a vaginal response. d. Instruct the woman to open her mouth and close her glottis, letting air escape after the push.

ANS: A The woman should maintain a side-lying position. The semi-Fowler position is the recommended side-lying position with a lateral tilt to the uterus. The Valsalva maneuver, which encourages the woman to hold her breath and tighten her abdominal muscles, should be avoided. Both the mouth and glottis should be open, letting air escape during the push.

Which interventions could a nurse apply to help stimulate contractions? (Select all that apply.) (3) a. Encouraging the patient to sit upright b. Assisting the patient to ambulate c. Stimulating the nipples d. Offering emotional support e. Allowing the patient to vent frustration

ANS: A, B, C Sitting upright, ambulating, and stimulating the nipples may encourage progression of labor. Offering emotional support and allowing patient to vent frustration are supportive to the patient but do not stimulate more effective labor.

Which are nonpharmacological forms of pain relief? (Select all that apply.) (3) a. Skin stimulation b. Diversion and distraction c. Breathing techniques d. Exercise e. Yoga

ANS: A, B, C Skin stimulation, diversion and distraction, and breathing techniques are the bases of nonpharmacological pain control. Although exercise and practices such as yoga and Pilates are beneficial, they are not means of pain control.

A woman is preparing for administration of a cervical ripening agent. What nursing actions will the nurse anticipate implementing? (Select all that apply.) (3) a. Insert IV. b. Record a baseline fetal heart rate. c. Explain procedure to patient. d. Instruct patient to ambulate immediately afterward. e. Ensure a tocolytic is available.

ANS: A, B, C The cervical ripening procedure should be explained to the woman and her family. A fetal heart rate baseline is recorded. An intravenous (IV) line with saline or heparin sodium (Hep-Lock) may be placed in case uterine tachysystole (hyperstimulation) occurs and IV tocolytics (drugs that reduce uterine contractions) are needed. After insertion of the prostaglandin gel, the woman remains on bed rest for 1 to 2 hours and is monitored for uterine contractions. Vital signs and fetal heart rate are also recorded.

While developing an intrapartum care plan for the patient in early labor, it is important that the nurse recognize that psychosocial factors may influence a womans experience of pain. These include (Select all that apply): (4) a. Culture. b. Anxiety and fear. c. Previous experiences with pain. d. Intervention of caregivers. e. Support systems.

ANS: A, B, C, E

What marks the end of the third stage of labor? a. Full cervical dilation b. Expulsion of the placenta and membranes c. Birth of the infant d. Engagement of the head

ANS: B The third stage of labor extends from the birth of the infant until the placenta is detached and expelled.

A new client and her partner arrive on the labor, delivery, recovery, and postpartum unit for the birth of their first child. You apply the electronic fetal monitor (EFM) to the woman. Her partner asks you to explain what is printing on the graph, referring to the EFM strip. He wants to know what the babys heart rate should be. Your best response is: a. Dont worry about that machine; thats my job. b. The top line graphs the babys heart rate. Generally the heart rate is between 110 and 160. The heart rate will fluctuate in response to what is happening during labor. c. The top line graphs the babys heart rate, and the bottom line lets me know how strong the contractions are. d. Your doctor will explain all of that later.

ANS: B The top line graphs the babys heart rate. Generally the heart rate is between 110 and 160. The heart rate will fluctuate in response to what is happening during labor educates the partner about fetal monitoring and provides support and information to alleviate his fears. Dont worry about that machine; thats my job discredits the partners feelings and does not provide the teaching he is requesting. The top line graphs the babys heart rate, and the bottom line lets me know how strong the contractions are provides inaccurate information and does not address the partners concerns about the fetal heart rate. The EFM graphs the frequency and duration of the contractions, not the intensity. Nurses should take every opportunity to provide client and family teaching, especially when information is requested.

The nerve block used in labor that provides anesthesia to the lower vagina and perineum is called: a. An epidural. b. A pudendal. c. A local. d. A spinal block.

ANS: B A pudendal block anesthetizes the lower vagina and perineum to provide anesthesia for an episiotomy and use of low forceps if needed. An epidural provides anesthesia for the uterus, perineum, and legs. A local provides anesthesia for the perineum at the site of the episiotomy. A spinal block provides anesthesia for the uterus, perineum, and down the legs.

With regard to a pregnant womans anxiety and pain experience, nurses should be aware that: a. Even mild anxiety must be treated. b. Severe anxiety increases tension, which increases pain, which in turn increases fear and anxiety, and so on. c. Anxiety may increase the perception of pain, but it does not affect the mechanism of labor. d. Women who have had a painful labor will have learned from the experience and have less anxiety the second time because of increased familiarity

ANS: B Anxiety and pain reinforce each other in a negative cycle. Mild anxiety is normal for a woman in labor and likely needs no special treatment other than the standard reassurances. Anxiety increases muscle tension and ultimately can build sufficiently to slow the progress of labor. Unfortunately, an anxious, painful first labor is likely to carry over, through expectations and memories, into an anxious and painful experience in the second pregnancy

The nurse coaches the primigravida not to bear down until the cervix is completely dilated. What may premature bearing down cause? a. Increased use of oxygen b. Cervical laceration c. Uterine rupture d. Compression of the cord

ANS: B Bearing down against a cervix that is not dilated can cause edema and laceration to the cervix.

It is important for the nurse to develop a realistic birth plan with the pregnant woman in her care. The nurse can explain that a major advantage of nonpharmacologic pain management is: a. Greater and more complete pain relief is possible. b. No side effects or risks to the fetus are involved. c. The woman remains fully alert at all times. d. A more rapid labor is likely.

ANS: B Because nonpharmacologic pain management does not include analgesics, adjunct drugs, or anesthesia, it is harmless to the mother and the fetus. There is less pain relief with nonpharmacologic pain management during childbirth. The womans alertness is not altered by medication; however, the increase in pain will decrease alertness. Pain management may or may not alter the length of labor. At times when pain is decreased, the mother relaxes and labor progresses at a quicker pace.

What assessment should be taken immediately after the anesthesiologist administers an epidural block to a laboring woman? a. Bladder for distention b. Blood pressure c. Sensation in the lower extremities d. Intravenous fluid flow rate

ANS: B Blood pressure is checked every 5 minutes when the epidural block is first begun. Bladder assessment is also important but not an initial assessment.

Which statement is the best rationale for assessing maternal vital signs between contractions? a. During a contraction, assessing fetal heart rates is the priority. b. Maternal circulating blood volume increases temporarily during contractions. c. Maternal blood flow to the heart is reduced during contractions. d. Vital signs taken during contractions are not accurate.

ANS: B During uterine contractions, blood flow to the placenta temporarily stops, causing a relative increase in the mothers blood volume, which in turn temporarily increases blood pressure and slows pulse. It is important to monitor fetal response to contractions; however, this question is concerned with the maternal vital signs. Maternal blood flow is increased during a contraction. Vital signs are altered by contractions but are considered accurate for that period of time.

With regard to systemic analgesics administered during labor, nurses should be aware that: a. Systemic analgesics cross the maternal blood-brain barrier as easily as they do the fetal blood-brain barrier. b. Effects on the fetus and newborn can include decreased alertness and delayed sucking. c. Intramuscular administration (IM) is preferred over intravenous (IV) administration. d. IV patient-controlled analgesia (PCA) results in increased use of an analgesic.

ANS: B Effects depend on the specific drug given, the dosage, and the timing. Systemic analgesics cross the fetal blood-brain barrier more readily than the maternal blood-brain barrier. IV administration is preferred over IM administration because the drug acts faster and more predictably. PCA results in decreased use of an analgesic.

The slight overlapping of cranial bones or shaping of the fetal head during labor is called: a. Lightening. b. Molding. c. Ferguson reflex. d. Valsalva maneuver.

ANS: B Fetal head formation is called molding. Molding also permits adaptation to various diameters of the maternal pelvis. Lightening is the mothers sensation of decreased abdominal distention, which usually occurs the week before labor. The Ferguson reflex is the contraction urge of the uterus after stimulation of the cervix. The Valsalva maneuver describes conscious pushing during the second stage of labor.

The role of the nurse with regard to informed consent is to: a. Inform the client about the procedure and have her sign the consent form. b. Act as a client advocate and help clarify the procedure and the options. c. Call the physician to see the client. d. Witness the signing of the consent form

ANS: B Nurses play a part in the informed consent process by clarifying and describing procedures or by acting as the womans advocate and asking the primary health care provider for further explanations. The physician is responsible for informing the woman of her options, explaining the procedure, and advising the client about potential risk factors. The physician must be present to explain the procedure to the client. However, the nurses responsibilities go further than simply asking the physician to see the client. The nurse may witness the signing of the consent form. However, depending on the states guidelines, the womans husband or another hospital health care employee may sign as witness.

What three measures should the nurse implement to provide intrauterine resuscitation? Select the response that best indicates the priority of actions that should be taken. a. Call the provider, reposition the mother, and perform a vaginal examination. b. Reposition the mother, increase intravenous (IV) fluid, and provide oxygen via face mask. c. Administer oxygen to the mother, increase IV fluid, and notify the care provider. d. Perform a vaginal examination, reposition the mother, and provide oxygen via face mask.

ANS: B Repositioning the mother, increasing intravenous (IV) fluid, and providing oxygen via face mask are correct nursing actions for intrauterine resuscitation. The nurse should initiate intrauterine resuscitation in an ABC manner, similar to basic life support. The first priority is to open the maternal and fetal vascular systems by repositioning the mother for improved perfusion. The second priority is to increase blood volume by increasing the IV fluid. The third priority is to optimize oxygenation of the circulatory volume by providing oxygen via face mask. If these interventions do not resolve the fetal heart rate issue quickly, the primary provider should be notified immediately.

During a strenuous labor, the woman asks for some pain remedy for the sudden pain between her scapulae that seems to occur with every breath she takes. What is the best nursing action? a. Give the pain remedy. b. Notify the charge nurse immediately. c. Turn the patient to her back and flex her knees. d. Suggest that the coach give her a back rub.

ANS: B Sudden pain between the scapulae during a strenuous labor is an indicator of uterine rupture. This should be reported immediately.

The nurse should be aware that an effective plan to achieve adequate pain relief without maternal risk is most effective if: a. The mother gives birth without any analgesic or anesthetic. b. The mother and familys priorities and preferences are incorporated into the plan. c. The primary health care provider decides the best pain relief for the mother and family. d. The nurse informs the family of all alternative methods of pain relief available in the hospital setting.

ANS: B The assessment of the woman, her fetus, and her labor is a joint effort of the nurse and the primary health care providers, who consult with the woman about their findings and recommendations. The needs of each woman are different, and many factors must be considered before a decision is made whether pharmacologic methods, nonpharmacologic methods, or a combination of the two will be used to manage labor pain.

During labor a fetus with an average heart rate of 135 beats/min over a 10-minute period would be considered to have: a. Bradycardia. b. A normal baseline heart rate. c. Tachycardia. d. Hypoxia.

ANS: B The baseline heart rate is measured over 10 minutes; a normal range is 110 to 160 beats/min. Bradycardia is a fetal heart rate (FHR) below 110 beats/min for 10 minutes or longer. Tachycardia is an FHR over 160 beats/min for 10 minutes or longer. Hypoxia is an inadequate supply of oxygen; no indication of this condition exists with a baseline heart rate in the normal range.

To teach patients about the process of labor adequately, the nurse knows that which event is the best indicator of true labor? a. Bloody show b. Cervical dilation and effacement c. Fetal descent into the pelvic inlet d. Uterine contractions every 7 minutes

ANS: B The conclusive distinction between true and false labor is that contractions of true labor cause progressive change in the cervix. Bloody show can occur before true labor. Fetal descent can occur before true labor. False labor may have contractions that occur this frequently; however, this is usually inconsistent.

The nurse has received report regarding her patient in labor. The womans last vaginal examination was recorded as 3 cm, 30%, and ?2-2. The nurses interpretation of this assessment is that: a. The cervix is effaced 3 cm, it is dilated 30%, and the presenting part is 2 cm above the ischial spines. b. The cervix is 3 cm dilated, it is effaced 30%, and the presenting part is 2 cm above the ischial spines. c. The cervix is effaced 3 cm, it is dilated 30%, and the presenting part is 2 cm below the ischial spines. d. The cervix is dilated 3 cm, it is effaced 30%, and the presenting part is 2 cm below the ischial spines.

ANS: B The correct description of the vaginal examination for this woman in labor is the cervix is 3 cm dilated, it is effaced 30%, and the presenting part is 2 cm above the ischial spines. The sterile vaginal examination is recorded as centimeters of cervical dilation, percentage of cervical dilation, and the relationship of the presenting part to the ischial spines (either above or below).

A woman in labor will receive general anesthesia prior to cesarean section. The nurse reminds the patient that food and fluids need to be restricted for several hours prior to delivery. What will this prevent? a. Nausea and vomiting b. Vomiting and aspiration c. Abdominal cramping d. Intestinal obstruction

ANS: B The major adverse effect of general anesthesia is aspiration of stomach contents.

As relates to fetal positioning during labor, nurses should be aware that: a. Position is a measure of the degree of descent of the presenting part of the fetus through the birth canal. b. Birth is imminent when the presenting part is at +4 to +5 cm below the spine. c. The largest transverse diameter of the presenting part is the suboccipitobregmatic diameter. d. Engagement is the term used to describe the beginning of labor.

ANS: B The station of the presenting part should be noted at the beginning of labor so that the rate of descent can be determined. Position is the relation of the presenting part of the fetus to the four quadrants of the mothers pelvis; station is the measure of degree of descent. The largest diameter usually is the biparietal diameter. The suboccipitobregmatic diameter is the smallest, although one of the most critical. Engagement often occurs in the weeks just before labor in nulliparas and before or during labor in multiparas.

Maternal hypotension is a potential side effect of regional anesthesia and analgesia. What nursing interventions could you use to raise the clients blood pressure (Select all that apply)? (3) a. Place the woman in a supine position. b. Place the woman in a lateral position. c. Increase intravenous (IV) fluids. d. Administer oxygen. e. Perform a vaginal examination.

ANS: B, C, D Nursing interventions for maternal hypotension arising from analgesia or anesthesia include turning the woman to a lateral position, increasing IV fluids, administering oxygen via face mask, elevating the womans legs, notifying the physician, administering an IV vasopressor, and monitoring the maternal and fetal status at least every 5 minutes until these are stable. Placing the client in a supine position would cause venous compression, thereby limiting blood flow to and oxygenation of the placenta and fetus. A sterile vaginal examination has no bearing on maternal blood pressure.

The nurse is providing a conference on nonpharmacological pain control methods. What major advantages of nonpharmacological pain control methods will the nurse include in the presentation? (Select all that apply.) (4) a. They sedate the mother. b. They do not slow labor. c. They do not dull the excitement of the birth experience. d. They do not have the potential to cause allergic reactions. e. They do not have to be delayed until labor is well established.

ANS: B, C, D, E All the options mentioned are benefits of nonpharmacological pain control methods with the exception of sedating the mother.

A tiered system of categorizing FHR has been recommended by regulatory agencies. Nurses, midwives, and physicians who care for women in labor must have a working knowledge of fetal monitoring standards and understand the significance of each category. These categories include (Select all that apply): (3) a. Reassuring. b. Category I. c. Category II. d. Nonreassuring. e. Category III.

ANS: B, C, E The three tiered system of FHR tracings include Category I, II, and III. Category I is a normal tracing requiring no action. Category II FHR tracings are indeterminate. This category includes tracings that do not meet Category I or III criteria. Category III tracings are abnormal and require immediate intervention.

A woman is 37 weeks pregnant and questioning the nurse about possible induction of labor at term. What conditions would contraindicate labor induction? (Select all that apply.) (2) a. Maternal gynecoid pelvis b. Placenta previa c. Horizontal cesarean incision d. Prolapsed cord e. Gestational diabetes

ANS: B, D Labor induction is contraindicated with placenta previa or a prolapsed umbilical cord. Gynecoid pelvis is the most favorable shape for vaginal delivery. Induction can be attempted as a VBAC after a horizontal cesarean incision but is contraindicated with a classic (vertical) incision. Gestational diabetes is not a contraindication for labor induction.

The nurse understands that the fetal head is in which of the following positions with a face presentation? A. Completely flexed B. Completely extended C. Partially extended D. Partially flexed

ANS: B. With a face presentation, the head is completely extended. With a vertex presentation, the head is completely or partially flexed. With a brow (forehead) presentation, the head would be partially extended.

When assessing the fetus using Leopold maneuvers, the nurse feels a round, firm, movable fetal part in the fundal portion of the uterus and a long, smooth surface in the mothers right side close to midline. What is the likely position of the fetus? a. ROA b. LSP c. RSA d. LOA

ANS: C The fetus is positioned anteriorly in the right side of the maternal pelvis with the sacrum as the presenting part. RSA is the correct three-letter abbreviation to indicate this fetal position. The first letter indicates the presenting part in either the right or left side of the maternal pelvis. The second letter indicates the anatomic presenting part of the fetus. The third letter stands for the location of the presenting part in relation to the anterior, posterior, or transverse portion of the maternal pelvis. Palpation of a round, firm fetal part in the fundal portion of the uterus would be the fetal head, indicating that the fetus is in a breech position with the sacrum as the presenting part in the maternal pelvis. Palpation of the fetal spine along the mothers right side denotes the location of the presenting part in the mothers pelvis. The ability to palpate the fetal spine indicates that the fetus is anteriorly positioned in the maternal pelvis.

The nurse arrives at the start of a shift on the labor unit to find a census of four patients in active labor. Which laboring patient should the nurse attend to first? a. 18-year-old primigravida with a fetal breech presentation b. 25-year-old multigravida with history of previous cesarean section c. 35-year-old multigravida with history of precipitate birth d. 16-year-old primigravida with a twin pregnancy

ANS: C A precipitate birth is completed in less than 3 hours. Labor often begins abruptly and intensifies quickly, rather than having a more subtle onset and gradual progression. Contractions may be frequent and intense, often from the onset. If the womans tissues do not yield easily to the powerful contractions, she may have uterine rupture, cervical lacerations, or hematoma. Fetal breech presentation, history of cesarean section, and multifetal pregnancy have associated risk factors, but not as immediate as precipitate birth.

What nursing assessment should be reported immediately after an amniotomy? a. Fetal heart rate is regular at 154 beats/min. b. Amniotic fluid is clear with flecks of vernix. c. Amniotic fluid is watery and pale green. d. Maternal temperature is 37.8 C.

ANS: C Amniotic fluid should be clear. Green fluid indicates the fetus has passed meconium, which is associated with fetal compromise.

A laboring woman received an opioid agonist (meperidine) intravenously 90 minutes before she gave birth. Which medication should be available to reduce the postnatal effects of Demerol on the neonate? a. Fentanyl (Sublimaze) b. Promethazine (Phenergan) c. Naloxone (Narcan) d. Nalbuphine (Nubain)

ANS: C An opioid antagonist can be given to the newborn as one part of the treatment for neonatal narcosis, which is a state of central nervous system (CNS) depression in the newborn produced by an opioid. Opioid antagonists such as naloxone (Narcan) can promptly reverse the CNS depressant effects, especially respiratory depression. Fentanyl, promethazine, and nalbuphine do not act as opioid antagonists to reduce the postnatal effects of Demerol on the neonate. Although meperidine (Demerol) is a low-cost medication and readily available, the use of Demerol in labor has been controversial because of its effects on the neonate.

According to standard professional thinking, nurses should auscultate the fetal heart rate (FHR): a. Every 15 minutes in the active phase of the first stage of labor in the absence of risk factors. b. Every 20 minutes in the second stage, regardless of whether risk factors are present. c. Before and after ambulation and rupture of membranes. d. More often in a womans first pregnancy.

ANS: C The FHR should be auscultated before and after administration of medications and induction of anesthesia. In the active phase of the first stage of labor, the FHR should be auscultated every 30 minutes if no risk factors are involved; with risk factors it should be auscultated every 15 minutes. In the second stage of labor the FHR should be auscultated every 15 minutes if no risk factors are involved; with risk factors it should be auscultated every 5 minutes. The fetus of a first-time mother is automatically at greater risk.

What is the function of contractions during the second stage of labor? a. Align the infant into the proper position for delivery b. Dilate and efface the cervix c. Push the infant out of the mothers body d. Separate the placenta from the uterine wall

ANS: C The contractions push the infant out of the mothers body as the second stage of labor ends with the birth of the infant.

The nurse is caring for a woman in the first stage of labor. What will the nurse remind the patient about contractions during this stage of labor? a. They get the infant positioned for delivery. b. They push the infant into the vagina. c. They dilate and efface the cervix. d. They get the mother prepared for true labor.

ANS: C The first stage of labor describes the time from the onset of labor until full dilation of the cervix.

A woman in labor has just received an epidural block. The most important nursing intervention is to: a. Limit parenteral fluids. b. Monitor the fetus for possible tachycardia. c. Monitor the maternal blood pressure for possible hypotension. d. Monitor the maternal pulse for possible bradycardia.

ANS: C The most important nursing intervention for a woman who has received an epidural block is to monitor the maternal blood pressure frequently for signs of hypotension. Intravenous fluids are increased for a woman receiving an epidural, to prevent hypotension. The nurse observes for signs of fetal bradycardia. The nurse monitors for signs of maternal tachycardia secondary to hypotension.

With regard to factors that affect how the fetus moves through the birth canal, nurses should be aware that: a. The fetal attitude describes the angle at which the fetus exits the uterus. b. Of the two primary fetal lies, the horizontal lie is that in which the long axis of the fetus is parallel to the long axis of the mother. c. The normal attitude of the fetus is called general flexion. d. The transverse lie is preferred for vaginal birth.

ANS: C The normal attitude of the fetus is general flexion. The fetal attitude is the relation of fetal body parts to one another. The horizontal lie is perpendicular to the mother; in the longitudinal (or vertical) lie the long axes of the fetus and the mother are parallel. Vaginal birth cannot occur if the fetus stays in a transverse lie.

The uterine contractions of a woman early in the active phase of labor are assessed by an internal uterine pressure catheter (IUPC). The nurse notes that the intrauterine pressure at the peak of the contraction ranges from 65 to 70 mm Hg and the resting tone range is 6 to 10 mm Hg. The uterine contractions occur every 3 to 4 minutes and last an average of 55 to 60 seconds. On the basis of this information, the nurse should: a. Notify the womans primary health care provider immediately. b. Prepare to administer an oxytocic to stimulate uterine activity. c. Document the findings because they reflect the expected contraction pattern for the active phase of labor. d. Prepare the woman for the onset of the second stage of labor.

ANS: C The nurse is responsible for monitoring the uterine contractions to ascertain whether they are powerful and frequent enough to accomplish the work of expelling the fetus and the placenta. In addition, the nurse would document these findings in the clients medical record. This labor pattern indicates that the client is in the active phase of the first stage of labor. Nothing indicates a need to notify the primary care provider at this time. Oxytocin augmentation is not needed for this labor pattern; this contraction pattern indicates adequate active labor. Her contractions eventually will become stronger, last longer, and come closer together during the transition phase of the first stage of labor. The transition phase precedes the second stage of labor, or delivery of the fetus.

The initial vaginal examination of a woman admitted to the labor unit reveals that the cervix is dilated 9 cm. The panicked woman begs the nurse, Please give me something. What is the most appropriate pain relief intervention for a woman in precipitate labor? a. Get an order for an intravenous narcotic. b. Notify the anesthesiologist for an epidural block. c. Stay and breathe with her during contractions. d. Tell her to bear with it because she is close to delivery.

ANS: C The nurse would stay with the woman experiencing precipitate labor and breathe with her during contractions to help the woman focus and cope with each contraction

A woman in the transition phase of labor requests a narcotic analgesic medication for pain relief. What should the nurse explain regarding giving a narcotic analgesic medication at this stage of labor? a. It can cause medication given at later stages to be ineffective. b. It will have no complications for the mother or infant. c. It may result in respiratory depression to the newborn. d. It will speed up labor and increase pain.

ANS: C The risk of narcotic analgesics is that they cross the placenta and can cause fetal respiratory depression

With regard to the turns and other adjustments of the fetus during the birth process, known as the mechanism of labor, nurses should be aware that: a. The seven critical movements must progress in a more or less orderly sequence. b. Asynclitism sometimes is achieved by means of the Leopold maneuver. c. The effects of the forces determining descent are modified by the shape of the womans pelvis and the size of the fetal head. d. At birth the baby is said to achieve restitution (i.e., a return to the C-shape of the womb).

ANS: C The size of the maternal pelvis and the ability of the fetal head to mold also affect the process. The seven identifiable movements of the mechanism of labor occur in combinations simultaneously, not in precise sequences. Asynclitism is the deflection of the babys head; the Leopold maneuver is a means of judging descent by palpating the mothers abdomen. Restitution is the rotation of the babys head after the infant is born.

What is an advantage of external electronic fetal monitoring? a. The ultrasound transducer can accurately measure short-term variability and beat-to-beat changes in the fetal heart rate. b. The tocotransducer can measure and record the frequency, regularity, intensity, and approximate duration of uterine contractions (UCs). c. The tocotransducer is especially valuable for measuring uterine activity during the first stage of labor. d. Once correctly applied by the nurse, the transducer need not be repositioned even when the woman changes positions.

ANS: C The tocotransducer is especially valuable for measuring uterine activity during the first stage of labor, particularly when the membranes are intact. Short-term changes cannot be measured with this technology. The tocotransducer cannot measure and record the intensity of UCs. The transducer must be repositioned when the woman or fetus changes position.

A woman in labor is breathing into a mouthpiece just before the start of her regular contractions. As she inhales, a valve opens, and gas is released. She continues to inhale the gas slowly and deeply until the contraction starts to subside. When the inhalation stops, the valve closes. This procedure is: a. Not used much anymore. b. Likely to be used in the second stage of labor but not in the first stage. c. An application of nitrous oxide. d. A prelude to cesarean birth

ANS: C This is an application of nitrous oxide, which could be used in either the first or second stage of labor (or both) as part of the preparation for a vaginal birth. Nitrous oxide is self-administered and found to be very helpful

The nurse providing care for the laboring woman should understand that late fetal heart rate (FHR)decelerations are the result of: a. Altered cerebral blood flow. b. Umbilical cord compression. c. Uteroplacental insufficiency. d. Meconium fluid.

ANS: C Uteroplacental insufficiency would result in late decelerations in the FHR. Altered fetal cerebral blood flow would result in early decelerations in the FHR. Umbilical cord compression would result in variable decelerations in the FHR. Meconium-stained fluid may or may not produce changes in the fetal heart rate, depending on the gestational age of the fetus and whether other causative factors associated with fetal distress are present.

A multiparous client who has been in labor for 2 hours states that she feels the urge to move her bowels. How should the nurse respond? A Let the client get up to use the potty B Allow the client to use a bedpan C Perform a pelvic examination D Check the fetal heart rate

ANS: C A complaint of rectal pressure usually indicates a low presenting fetal part, signaling imminent delivery. The nurse should perform a pelvic examination to assess the dilation of the cervix and station of the presenting fetal part.

The baseline fetal heart rate (FHR) is the average rate during a 10-minute segment. Changes in FHR are categorized as periodic or episodic. These patterns include both accelerations and decelerations. The labor nurse is evaluating the patients most recent 10-minute segment on the monitor strip and notes a late deceleration. This is likely to be caused by which physiologic alteration (Select all that apply)? (2) a. Spontaneous fetal movement b. Compression of the fetal head c. Placental abruption d. Cord around the babys neck e. Maternal supine hypotension

ANS: C, E Late decelerations are almost always caused by uteroplacental insufficiency. Insufficiency is caused by uterine tachysystole, maternal hypotension, epidural or spinal anesthesia, IUGR, intraamniotic infection, or placental abruption. Spontaneous fetal movement, vaginal examination, fetal scalp stimulation, fetal reaction to external sounds, uterine contractions, fundal pressure and abdominal palpation are all likely to cause accelerations of the FHR. Early decelerations are most often the result of fetal head compression and may be caused by uterine contractions, fundal pressure, vaginal examination, and placement of an internal electrode. A variable deceleration is likely caused by umbilical cord compression. This may happen when the umbilical cord is around the babys neck, arm, leg, or other body part or when there is a short cord, a knot in the cord, or a prolapsed cord.

An 18-year-old pregnant woman, gravida 1, is admitted to the labor and birth unit with moderate contractions every 5 minutes that last 40 seconds. The woman states, My contractions are so strong that I dont know what to do with myself. The nurse should: a. Assess for fetal well-being. b. Encourage the woman to lie on her side. c. Disturb the woman as little as possible. d. Recognize that pain is personalized for each individual.

ANS: D Each womans pain during childbirth is unique and is influenced by a variety of physiologic, psychosocial, and environmental factors. A critical issue for the nurse is how support can make a difference in the pain of the woman during labor and birth. Assessing for fetal well-being includes no information that would indicate fetal distress or a logical reason to be overly concerned about the well-being of the fetus. The left lateral position is used to alleviate fetal distress, not maternal stress. The nurse has an obligation to provide physical, emotional, and psychosocial care and support to the laboring woman. This client clearly needs support.

Why should the nurse encourage the mother to void during the fourth stage of labor? a. A full bladder could interfere with cervical dilation. b. A full bladder could obstruct progress of the infant through the birth canal. c. A full bladder could obstruct the passage of the placenta. d. A full bladder could predispose the mother to uterine hemorrhage.

ANS: D A full bladder immediately after birth can cause excessive bleeding because it pushes the uterus upward and interferes with contractions.

Which basic type of pelvis includes the correct description and percentage of occurrence in women? a. Gynecoid: classic female pelvis; heart shaped; 75% b. Android: resembling the male pelvis; wide oval; 15% c. Anthropoid: resembling the pelvis of the ape; narrow; 10% d. Platypelloid: flattened, wide, and shallow pelvis; 3%

ANS: D A platypelloid pelvis is flattened, wide, and shallow; approximately 3% of women have this shape. The gynecoid pelvis is the classic female shape, slightly ovoid and rounded; approximately 50% of women have this shape. An android or malelike pelvis is heart shaped; approximately 23% of women have this shape. An anthropoid or apelike pelvis is oval and wide; approximately 24% of women have this shape.

A woman in labor has had an epidural block for pain relief. The nurse will be assessing carefully for what associated side effect of this type of regional anesthesia? a. Reduced fetal heart rate b. Long, intense contractions c. Sudden leg cramps d. Bladder distention

ANS: D A side effect of an epidural block is urine retention because the anesthesia interferes with the womans ability to have an urge to void. The patient may have to be catheterized.

When assessing the relative advantages and disadvantages of internal and external electronic fetal monitoring, nurses comprehend that both: a. Can be used when membranes are intact. b. Measure the frequency, duration, and intensity of uterine contractions. c. May need to rely on the woman to indicate when uterine activity (UA) is occurring. d. Can be used during the antepartum and intrapartum periods.

ANS: D External monitoring can be used in both periods; internal monitoring can be used only in the intrapartum period. For internal monitoring the membranes must have ruptured, and the cervix must be sufficiently dilated. Internal monitoring measures the intensity of contractions; external monitoring cannot do this. With external monitoring, the woman may need to alert the nurse that UA is occurring; internal monitoring does not require this.

Several hours into labor, a woman complains of dizziness, numbness, and tingling of her hands and mouth. What does the nurse recognize these symptoms signify? a. Hypertension b. Anxiety c. Anoxia d. Hyperventilation

ANS: D Hyperventilation is sometimes a problem if a woman is breathing rapidly

The nurse formulates a nursing diagnosis for a woman in the fourth stage of labor. What is the most appropriate nursing diagnosis? a. Pain related to increasing frequency and intensity of contractions. b. Fear related to the probable need for cesarean delivery. c. Dysuria related to prolonged labor and decreased intake. d. Risk for injury related to hemorrhage.

ANS: D In the fourth stage of labor, a priority nursing action is identifying and preventing hemorrhage

At a prenatal visit, a primigravida asks the nurse how she will know her labor has started. The nurse knows that what indicates the beginning of true labor? a. Contractions that are relieved by walking b. Discomfort in the abdomen and groin c. A decrease in vaginal discharge d. Regular contractions becoming more frequent and intense

ANS: D In true labor, contractions gradually develop a regular pattern and become more frequent, longer, and more intense.

Which fetal heart rate (FHR) finding would concern the nurse during labor? a. Accelerations with fetal movement b. Early decelerations c. An average FHR of 126 beats/min d. Late decelerations

ANS: D Late decelerations are caused by uteroplacental insufficiency and are associated with fetal hypoxemia. They are considered ominous if persistent and uncorrected. Accelerations in the FHR are an indication of fetal wellbeing. Early decelerations in the FHR are associated with head compression as the fetus descends into the maternal pelvic outlet; they generally are not a concern during normal labor

Which method of pain management is safest for a gravida 3 para 2 admitted at 8 cm cervical dilation? a. Epidural anesthesia b. Narcotics c. Spinal block d. Breathing and relaxation techniques

ANS: D Nonpharmacologic methods of pain management may be the best option for a woman in advanced labor. It is unlikely that enough time remains to administer epidural or spinal anesthesia. A narcotic given at this time may reach its peak about the time of birth and result in respiratory depression in the newborn.

As a perinatal nurse you realize that a fetal heart rate that is tachycardic, is bradycardic, or has late decelerations or loss of variability is nonreassuring and is associated with: a. Hypotension. b. Cord compression. c. Maternal drug use. d. Hypoxemia.

ANS: D Nonreassuring heart rate patterns are associated with fetal hypoxemia. Fetal bradycardia may be associated with maternal hypotension. Fetal variable decelerations are associated with cord compression. Maternal drug use is associated with fetal tachycardia.

While evaluating an external monitor tracing of a woman in active labor whose labor is being induced, the nurse notes that the fetal heart rate (FHR) begins to decelerate at the onset of several contractions and returns to baseline before each contraction ends. The nurse should: a. Change the womans position. b. Discontinue the oxytocin infusion. c. Insert an internal monitor. d. Document the finding in the clients record.

ANS: D The FHR indicates early decelerations, which are not an ominous sign and do not require any intervention. The nurse should simply document these findings.

In order to evaluate the condition of the patient accurately during labor, the nurse should be aware that: a. The womans blood pressure will increase during contractions and fall back to prelabor normal between contractions. b. Use of the Valsalva maneuver is encouraged during the second stage of labor to relieve fetal hypoxia. c. Having the woman point her toes will reduce leg cramps. d. The endogenous endorphins released during labor will raise the womans pain threshold and produce sedation.

ANS: D The endogenous endorphins released during labor will raise the woman's pain threshold and produce sedation. In addition, physiologic anesthesia of the perineal tissues, caused by the pressure of the presenting part, decreases the mothers perception of pain. Blood pressure increases during contractions but remains somewhat elevated between them. Use of the Valsalva maneuver is discouraged during second stage labor because of a number of unhealthy outcomes, including fetal hypoxia. Pointing the toes can cause leg cramps, as can the process of labor itself.

The factors that affect the process of labor and birth, known commonly as the five Ps, include all except: a. Passenger. b. Passageway. c. Powers. d. Pressure

ANS: D The five Ps are passenger (fetus and placenta), passageway (birth canal), powers (contractions), position of the mother, and psychologic response.

What does the nurse note when measuring the frequency of a laboring womans contractions? a. How long the patient states the contractions last b. The time between the end of one contraction and the beginning of the next c. The time between the beginning and the end of one contraction d. The time between the beginning of one contraction and the beginning of the next

ANS: D The frequency of contractions is the elapsed time from the beginning of one contraction to the beginning of the next contraction.

A patient who received an epidural block asks why her blood pressure is taken so often. What is the nurses best response to explain the frequent blood pressure assessments? a. They ensure that unsafe levels of hypertension do not occur. b. They help assess for the need for further pain relief. c. They monitor the progress of labor. d. They ensure adequate placental perfusion.

ANS: D The hypotension that accompanies an epidural block may cause inadequate perfusion of the placenta, leading to fetal hypoxia.

A primigravida at 39 weeks of gestation is observed for 2 hours in the intrapartum unit. The fetal heart rate has been normal. Contractions are 5 to 9 minutes apart, 20 to 30 seconds in duration, and of mild intensity. Cervical dilation is 1 to 2 cm and uneffaced (unchanged from admission). Membranes are intact. The nurse should expect the woman to be: a. Admitted and prepared for a cesarean birth. b. Admitted for extended observation. c. Discharged home with a sedative. d. Discharged home to await the onset of true labor.

ANS: D This situation describes a woman with normal assessments who is probably in false labor and will probably not deliver rapidly once true labor begins. These are all indications of false labor without fetal distress. There is no indication that further assessment or cesarean birth is indicated. The patient will likely be discharged; however, there is no indication that a sedative is needed.

You are evaluating the fetal monitor tracing of your client, who is in active labor. Suddenly you see the fetal heart rate (FHR) drop from its baseline of 125 down to 80. You reposition the mother, provide oxygen, increase intravenous (IV) fluid, and perform a vaginal examination. The cervix has not changed. Five minutes have passed, and the fetal heart rate remains in the 80s. What additional nursing measures should you take? a. Scream for help.8 b. Insert a Foley catheter. c. Start Pitocin. d. Notify the care provider immediately.

ANS: D To relieve an FHR deceleration, the nurse can reposition the mother, increase IV fluid, and provide oxygen. If oxytocin is infusing, it should be discontinued. If the FHR does not resolve, the primary care provider should be notified immediately. Inserting a Foley catheter is an inappropriate nursing action. If the FHR were to continue in a nonreassuring pattern, a cesarean section could be warranted, which would require a Foley catheter. However, the physician must make that determination. Pitocin may place additional stress on the fetus.

What is the most appropriate statement from the nurse when coaching the laboring woman with a fully dilated cervix to push? a. At the beginning of a contraction, hold your breath and push for 10 seconds. b. Take a deep breath and push between contractions. c. Begin pushing when a contraction starts and continue for the duration of the contraction. d. At the beginning of a contraction, take two deep breaths and push with the second exhalation.

ANS: D When the cervix is fully dilated, the woman should take a deep breath and exhale at the beginning of a contraction, and then take another deep breath and push while exhaling.

What are the rationales for labor induction? (Select all that apply.) (2) a. Placenta previa b. Prolapse of cord c. High station of fetus d. Maternal diabetes e. Placental insufficiency

ANS: D, E Maternal diabetes and placental insufficiency are rationales for induction. The other options are contraindications for labor induction.

Which of the following is a priority when caring for a woman during the fourth stage of labor? A) Assessing the uterine fundus B) Offering fluids as indicated C) Encouraging the woman to void D) Assisting with perineal care

Ans. A Assessing the uterine fundus During the fourth stage of labor, a priority is to assess the woman's fundus to prevent postpartum hemorrhage. Offering fluids, encouraging voiding, and assisting with perineal care are important but not an immediate priority. (pg. 483)

A woman has just entered the second stage of labor. The nurse would focus care on which of the following? A) Encouraging the woman to push when she has a strong desire to do so B) Alleviating perineal discomfort with the application of ice packs C) Palpating the woman's fundus for position and firmness D) Completing the identification process of the newborn with the mother

Ans. A During the second stage of labor, nursing interventions focus on motivating the woman, encouraging her to put all her efforts toward pushing. Alleviating perineal discomfort with ice packs and palpating the woman's fundus would be appropriate during the fourth stage of labor. Completing the newborn identification process would be appropriate during the third stage of labor. (pg. 475)

When applying the ultrasound transducers for continuous external electronic fetal monitoring, at which location would the nurse place the transducer to record the FHR? A) Over the uterine fundus where contractions are most intense B) Above the umbilicus toward the right side of the diaphragm C) Between the umbilicus and the symphysis pubis D) Between the xiphoid process and umbilicus

Ans. C Between the umbilicus and the symphysis pubis The ultrasound transducer is positioned on the maternal abdomen in the midline between the umbilicus and the symphysis pubis. The tocotransducer is placed over the uterine fundus in the area of greatest contractility.

The nurse notes persistent early decelerations on the fetal monitoring strip. Which of the following would the nurse do next? A) Continue to monitor the FHR because this pattern is benign. B) Perform a vaginal exam to assess cervical dilation and effacement. C) Stay with the client while reporting the finding to the physician. D) Administer oxygen after turning the client on her left side

Ans. A Early decelerations are not indicative of fetal distress and do not require intervention. Therefore, the nurse would continue to monitor the fetal heart rate pattern. They are most often seen during the active stage of any normal labor, during pushing, crowning, or vacuum extraction. They are thought to be a result of fetal head compression that results in a reflex vagal response with a resultant slowing of the FHR during uterine contractions. There is no need to perform a vaginal exam, report the finding to the physician, or administer oxygen. (pg. 455)

The nurse is performing Leopold's maneuvers to determine fetal presentation, position, and lie. Which action would the nurse do first? A) Feel for the fetal buttocks or head while palpating the abdomen. B) Feel for the fetal back and limbs as the hands move laterally on the abdomen. C) Palpate for the presenting part in the area just above the symphysis pubis. D) Determine flexion by pressing downward toward the symphysis pubis.

Ans. A The first maneuver involves feeling for the buttocks and head. Next the nurse palpates on which side the fetal back is located. The third maneuver determines presentation and involves palpating the area just above the symphysis pubis. The final maneuver determines attitude and involves applying downward pressure in the direction of the symphysis pubis. (pg. 448

A nurse is describing the different types of regional analgesia and anesthesia for labor to a group of pregnant women. Which statement by the group indicates that the teaching was successful? A) "We can get up and walk around after receiving combined spinal-epidural analgesia." B) "Higher anesthetic doses are needed for patient-controlled epidural analgesia." C) "A pudendal nerve block is highly effective for pain relief in the first stage of labor." D) "Local infiltration using lidocaine is an appropriate method for controlling contraction pain."

Ans. A When compared with traditional epidural or spinal analgesia, which often keeps the woman lying in bed, combined spinal-epidural analgesia allows the woman to ambulate ("walking epidural"). Patient-controlled epidural analgesia provides equivalent analgesia with lower anesthetic use, lower rates of supplementation, and higher client satisfaction. Pudendal nerve blocks are used for the second stage of labor, an episiotomy, or an operative vaginal birth with outlet forceps or vacuum extractor. Local infiltration using lidocaine does not alter the pain of uterine contractions, but it does numb the immediate area of the episiotomy or laceration. (pg. 467)

A client's membranes spontaneously ruptured, as evidenced by a gush of clear fluid with a contraction. Which of the following would the nurse do next? A) Check the fetal heart rate. B) Perform a vaginal exam. C) Notify the physician immediately. D) Change the linen saver pad

Ans. A When membranes rupture, the priority focus is on assessing fetal heart rate first to identify a deceleration, which might indicate cord compression secondary to cord prolapse. A vaginal exam may be done later to evaluate for continued progression of labor. The physician should be notified, but this is not a priority at this time. Changing the linen saver pad would be appropriate once the fetal status is determined and the physician has been notified. (pg. 446)

A nurse palpates a woman's fundus to determine contraction intensity. Which of the following would be most appropriate for the nurse to use for palpation? A) Finger pads B) Palm of the hand C) Finger tips D) Back of the hand

Ans. A Finger pads To palpate the fundus for contraction intensity, the nurse would place the pads of the fingers on the fundus and describe how it feels. Using the finger tips, palm, or back of the hand would be inappropriate. (pg. 447)

A woman in labor received an opioid close to the time of birth. The nurse would assess the newborn for which of the following? A) Respiratory depression B) Urinary retention C) Abdominal distention D) Hyperreflexia

Ans. A Respiratory depression Opioids given close to the time of birth can cause central nervous system depression, including respiratory depression, in the newborn, necessitating the administration of naloxone. Urinary retention may occur in the woman who received neuraxial opioids. Abdominal distention is not associated with opioid administration. Hyporeflexia would be more commonly associated with central nervous system depression due to opioids. (pg. 464)

After teaching a group of students about fetal heart rate patterns, the instructor determines the need for additional teaching when the students identify which of the following as indicating normal fetal acid- base status? (Select all that apply.) (3) A) Sinusoidal pattern B) Recurrent variable decelerations C) Fetal bradycardia D) Absence of late decelerations E) Moderate baseline variability

Ans. A,B,C Predictors of normal fetal acid-base status include a baseline rate between 110 and 160 bpm, moderate baseline variability, and absences of later or variable decelerations. Sinusoidal pattern, recurrent variable decelerations, and fetal bradycardia are predictive of abnormal fetal acid-base status. (pg. 453)

Which of the following would be most appropriate for the nurse to suggest about pushing to a woman in the second stage of labor? A) "Lying flat with your head elevated on two pillows makes pushing easier." B) "Choose whatever method you feel most comfortable with for pushing." C) " Let me help you decide when it is time to start pushing." D) "Bear down like you're having a bowel movement with every contraction."

Ans. B "Choose whatever method you feel most comfortable with for pushing." The role of the nurse should be to support the woman in her choice of pushing method and to encourage confidence in her maternal instinct of when and how to push. In the absence of any complications, nurses should not be controlling this stage of labor, but empowering women to achieve a satisfying experience. Common practice in many labor units is still to coach women to use closed glottis pushing with every contraction, starting at 10 cm of dilation, a practice that is not supported by research. Research suggests that directed pushing during the second stage may be accompanied by a significant decline in fetal pH and may cause maternal muscle and nerve damage if done too early. Effective pushing can be achieved by assisting the woman to assume a more upright or squatting position. Supporting spontaneous pushing and encouraging women to choose their own method of pushing should be accepted as best clinical practice. (pg. 475)

After describing continuous internal electronic fetal monitoring to a laboring woman and her partner, which of the following would indicate the need for additional teaching? A) "This type of monitoring is the most accurate method for our baby." B) "Unfortunately, I'm going to have to stay quite still in bed while it is in place." C) "This type of monitoring can only be used after my membranes rupture." D) "You'll be inserting a special electrode into my baby's scalp."

Ans. B "Unfortunately, I'm going to have to stay quite still in bed while it is in place." With continuous internal electronic monitoring, maternal position changes and movement do not interfere with the quality of the tracing. Continuous internal monitoring is considered the most accurate method, but it can be used only if certain criteria are met, such as rupture of membranes. A spiral electrode is inserted into the fetal presenting part, usually the head.

A woman's amniotic fluid is noted to be cloudy. The nurse interprets this finding as which of the following? A) Normal B) Possible infection C) Meconium passage D) Transient fetal hypoxia

Ans. B Amniotic fluid should be clear when the membranes rupture, either spontaneously or artificially through an amniotomy (a disposable plastic hook [Amnihook] is used to perforate the amniotic sac). Cloudy or foul-smelling amniotic fluid indicates infection. Green fluid may indicate that the fetus has passed meconium secondary to transient hypoxia, prolonged pregnancy, cord compression, intrauterine growth restriction, maternal hypertension, diabetes, or chorioamnionitis; however, it is considered a normal occurrence if the fetus is in a breech presentation. (pg. 447)

A woman in labor is to receive continuous internal electronic fetal monitoring. The nurse reviews the woman's medical record to ensure which of the following as being required? A) Intact membranes B) Cervical dilation of 2 cm or more C) Floating presenting fetal part D) A neonatologist to insert the electrode

Ans. B Cervical dilation of 2 cm or more For continuous internal electronic fetal monitoring, four criteria must be met: ruptured membranes, cervical dilation of at least 2 cm, fetal presenting part low enough to allow placement of the electrode, and a skilled practitioner available to insert the electrode

A woman in labor is to receive continuous internal electronic fetal monitoring. The nurse reviews the woman's medical record to ensure which of the following as being required? A) Intact membranes B) Cervical dilation of 2 cm or more C) Floating presenting fetal part D) A neonatologist to insert the electrode

Ans. B For continuous internal electronic fetal monitoring, four criteria must be met: ruptured membranes, cervical dilation of at least 2 cm, fetal presenting part low enough to allow placement of the electrode, and a skilled practitioner available to insert the electrode. (pg. 452)

A nurse is assessing a woman after birth and notes a second-degree laceration. The nurse interprets this as indicating that the tear extends through which of the following? A) Skin B) Muscles of perineal body C) Anal sphincter D) Anterior rectal wall

Ans. B Muscle of perineal body The extent of the laceration is defined by depth: a first-degree laceration extends through the skin; a second-degree laceration extends through the muscles of the perineal body; a third-degree laceration continues through the anal sphincter muscle; and a fourth-degree laceration also involves the anterior rectal wall. (pg. 475)

A nurse is reviewing the fetal heart rate pattern and observes abrupt decreases in FHR below the baseline, appearing as a U-shape. The nurse interprets these changes as reflecting which of the following? A) Early decelerations B) Variable decelerations C) Prolonged decelerations D) Late decelerations

Ans. B Variable decelerations Variable decelerations present as visually apparent abrupt decreases in FHR below baseline and have an unpredictable shape on the FHR baseline, possibly demonstrating no consistent relationship to uterine contractions. The shape of variable decelerations may be U, V, or W, or they may not resemble other patterns. Early decelerations are visually apparent, usually symmetrical and characterized by a gradual decrease in the FHR in which the nadir (lowest point) occurs at the peak of the contraction. They are thought to be a result of fetal head compression that results in a reflex vagal response with a resultant slowing of the FHR during uterine contractions. Late decelerations are visually apparent, usually symmetrical, transitory decreases in FHR that occur after the peak of the contraction. The FHR does not return to baseline levels until well after the contraction has ended. Delayed timing of the deceleration occurs, with the nadir of the uterine contraction. Late decelerations are associated with uteroplacental insufficiency. Prolonged decelerations are abrupt FHR declines of at least 15 bpm that last longer than 2 minutes but less than 10 minutes. (pg. 455)

A pregnant woman admitted to the labor and birth suite undergoes rapid HIV testing and is found to be HIV-positive. Which of the following would the nurse expect to include when developing a plan of care for this women? (Select all that apply.) (3) A) Administration of penicillin G at the onset of labor B) Avoidance of scalp electrodes for fetal monitoring C) Refraining from obtaining fetal scalp blood for pH testing D) Adminstering zidovudine at the onset of labor. E) Electing for the use of forceps-assisted delivery

Ans. B,C,D' To reduce perinatal transmission, HIV-positive women are given zidovudine (ZDV) (2mg/kg IV over an hour, and then a maintenance infusion of 1 mg/kg per hour until birth) or a single 200-mg oral dose of nevirapine at the onset of labor; the newborn is given ZDV orally (2 mg/kg body weight every 6 hours) and should be continued for 6 weeks (Gardner, Carter, Enzman-Hines, & Hernandez, 2011). To further reduce the risk of perinatal transmission, ACOG and the U.S. Public Health Service recommend that HIV-infected women with plasma viral loads of more than 1,000 copies per milliliter be counseled regarding the benefits of elective cesarean birth (Reshi & Lone, 2010). Additional interventions to reduce the transmission risk would include avoiding use of scalp electrode for fetal monitoring or doing a scalp blood sampling for fetal pH, delaying amniotomy, encouraging formula feeding after birth, and avoiding invasive procedure such as forceps or vacuum-assisted devices. (pg. 473)

A nurse is completing the assessment of a woman admitted to the labor and birth suite. Which of the following would the nurse expect to include as part of the physical assessment? (Select all that apply.) (3) A) Current pregnancy history B) Fundal height measurement C) Support system D) Estimated date of birth E) Membrane status F) Contraction pattern

Ans. B,E,F As part of the admission physical assessment, the nurse would assess fundal height, membrane status and contractions. Current pregnancy history, support systems, and estimated date of birth would be obtained when collecting the maternal health history. (pg. 471)

A nurse is assisting with the delivery of a newborn. The fetal head has just emerged. Which of the following would be done next? A) Suctioning of the mouth and nose B) Clamping of the umbilical cord C) Checking for the cord around the neck D) Drying of the newborn

Ans. C Checking for the cord around the neck once the fetal head has emerged, the primary care provider explores the fetal neck to see if the umbilical cord is wrapped around it. If it is, the cord is slipped over the head to facilitate delivery. Then the health care provider suctions the newborn's mouth first (because the newborn is an obligate nose breather) and then the nares with a bulb syringe to prevent aspiration of mucus, amniotic fluid, or meconium. Finally the umbilical cord is double-clamped and cut between the clamps. The newborn is placed under the radiant warmer, dried, assessed, wrapped in warm blankets and placed on the woman's abdomen for warmth and closeness. (pg. 480)

When planning the care of a woman in the active phase of labor, the nurse would anticipate assessing the fetal heart rate at which interval? A) Every 2 to 4 hours B) Every 45 to 60 minutes C) Every 15 to 30 minutes D) Every 10 to 15 minutes

Ans. C Every 15 to 30 minutes During the active phase of labor, FHR is monitored every 15 to 30 minutes. FHR is assessed every 30 to 60 minutes during the latent phase of labor. The woman's temperature is typically assessed every 4 hours during the first stage of labor and every 2 hours after ruptured membranes. Blood pressure, pulse, and respirations are assessed every hour during the latent phase and every 30 minutes during the active and transition phases. Contractions are assessed every 30 to 60 minutes during the latent phase and every 15 to 30 minutes during the active phase, and every 15 minutes during transition. (pg. 473)

When palpating the fundus during a contraction, the nurse notes that it feels like a chin. The nurse interprets this finding as indicating which type of contraction? A) Intense B) Strong C) Moderate D) Mild

Ans. C Moderate A contraction that feels like the chin typically represents a moderate contraction. A contraction described as feeling like the tip of the nose indicates a mild contraction. A strong contraction feels like the forehead. (pg. 447)

A woman in labor who received an opioid for pain relief develops respiratory depression. The nurse would expect which agent to be administered? A) Butorphanol B) Fentanyl C) Naloxone D) Promethazine

Ans. C Naloxone is an opioid antagonist used to reverse the effects of opioids such as respiratory depression. Butorphanol and fentanyl are opioids and would cause further respiratory depression. Promethazine is an ataractic used as an adjunct to potentiate the effectiveness of the opioid. (pg. 464)

When assessing fetal heart rate, the nurse finds a heart rate of 175 bpm, accompanied by a decrease in variability and late decelerations. Which of the following would the nurse do next? A) Have the woman change her position. B) Administer oxygen. C) Notify the health care provider. D) Continue to monitor the pattern every 15 minutes

Ans. C Notify the health care provider. Fetal tachycardia as evidenced by a fetal heart rate greater than 160 bpm accompanied by a decrease in variability and late decelerations is an ominous sign indicating the need for prompt intervention. The health care provider should be notified immediately and then measures should be instituted such as having the woman lie on her side and administering oxygen. In this instance, monitoring should be continuous to detect any further changes and evaluate the effectiveness of interventions. (pg. 453)

A nurse is providing care to a woman during the third stage of labor. Which of the following would alert the nurse that the placenta is separating? (Select all that apply.) (1) A) Boggy, soft uterus B) Uterus becoming discoid shaped C) Sudden gush of dark blood from the vagina D) Shortening of the umbilical cord

Ans. C Sudden gush of dark blood from the vagina Signs that the placenta is separating include a firmly contracting uterus, a change in uterine shape from discoid to globular ovoid, a sudden gush of dark blood from the vaginal opening, and lengthening of the umbilical cord protruding from the vagina. (pg. 482)

A woman is admitted to the labor and birthing suite. Vaginal examination reveals that the presenting part is approximately 2 cm above the ischial spines. The nurse documents this finding as: A) +2 station B) 0 station C) -2 station D) Crowning

Ans. C The ischial spines serve as landmarks and are designated as zero status. If the presenting part is palpated higher than the maternal ischial spines, a negative number is assigned. Therefore, the nurse would document the finding as -2 station. If the presenting part is below the ischial spines, then the station would be +2. Crowning refers to the appearance of the fetal head at the vaginal opening. (pg. 446)

A nurse is explaining the use of therapeutic touch as a pain relief measure during labor. Which of the following would the nurse include in the explanation? A) "This technique focuses on manipulating body tissues." B) "The technique requires focusing on a specific stimulus." C) "This technique redirects energy fields that lead to pain." D) "The technique involves light stroking of the abdomen with breathing."

Ans. C Therapeutic touch is an energy therapy and is based on the premise that the body contains energy fields that lead to either good or ill health and that the hands can be used to redirect the energy fields that lead to pain. Attention focusing and imagery involve focusing on a specific stimulus. Massage focuses on manipulating body tissues. Effleurage involves light stroking of the abdomen in rhythm with breathing. (pg. 462)

A client states, "I think my waters broke! I felt this gush of fluid between my legs." The nurse tests the fluid with Nitrazine paper and confirms membrane rupture if the paper turns: A) Yellow B) Olive green C) Pink D) Blue

Ans. D Amniotic fluid is alkaline and turns Nitrazine paper blue. Nitrazine paper that remains yellow to olive green suggests that the membranes are most likely intact. (pg. 446)

A woman in labor has chosen to use hydrotherapy as a method of pain relief. Which statement by the woman would lead the nurse to suspect that the woman needs additional teaching? A) "The warmth and buoyancy of the water has a nice relaxing effect." B) "I can stay in the bath for as long as I feel comfortable." C) "My cervix should be dilated more than 5 cm before I try using this method." D) "The temperature of the water should be at least 105° F."

Ans. D "The temperature of the water should be at least 105°F." Hydrotherapy is an effective pain relief method. The water temperature should not exceed body temperature. Therefore, a temperature of 105° F would be too warm. The warmth and buoyancy have a relaxing effect and women are encouraged to stay in the bath as long as they feel comfortable. The woman should be in active labor with cervical dilation greater than 5 cm. (pg. 458)

During a follow-up prenatal visit, a pregnant woman asks the nurse, "How long do you think I will be in labor?" Which response by the nurse would be most appropriate? A) "It's difficult to predict how your labor will progress, but we'll be there for you the entire time." B) "Since this is your first pregnancy, you can estimate it will be about 10 hours." C) "It will depend on how big the baby is when you go into labor." D) "Time isn't important; your health and the baby's health are key."

Ans: A Feedback: It is difficult to predict how a labor will progress and therefore equally difficult to determine how long a woman's labor will last. There is no way to estimate the likely strength and frequency of uterine contractions, the extent to which the cervix will soften and dilate, and how much the fetal head will mold to fit the birth canal. We cannot know beforehand whether the complex fetal rotations needed for an efficient labor will take place properly. All of these factors are unknowns when a woman starts labor. Telling the woman an approximate time would be inappropriate because there is no way to determine the length of labor. It is highly individualized. Although fetal size and maternal and fetal health are important considerations, these responses do not address the woman's concern.

Which of the following would indicate to the nurse that the placenta is separating? A) Uterus becomes globular B) Fetal head is at vaginal opening C) Umbilical cord shortens D) Mucous plug is expelled

Ans: A Feedback: Placental separation is indicated by the uterus changing shape to globular and upward rising of the uterus. Additional signs include a sudden trickle of blood from the vaginal opening, and lengthening (not shortening) of the umbilical cord. The fetal head at the vaginal opening is termed crowning and occurs before birth of the head. Expulsion of the mucous plug is a premonitory sign of labor.

A nurse is describing how the fetus moves through the birth canal. Which of the following would the nurse identify as being most important in allowing the fetal head to move through the pelvis? A) Sutures B) Fontanelles C) Frontal bones D) Biparietal diameter

Ans: A Feedback: Sutures are important because they allow the cranial bones to overlap in order for the head to adjust in shape (elongate) when pressure is exerted on it by uterine contractions or the maternal bony pelvis. Fontanelles are the intersections formed by the sutures. The frontal bones, along with the parietal and occipital bones are bones of the cranium that are soft and pliable. The biparietal diameter is an important diameter that can affect the birth process.

The fetus of a nulliparous woman is in a shoulder presentation. The nurse would most likely prepare the client for which type of birth? A) Cesarean B) Vaginal C) Forceps-assisted D) Vacuum extraction

Ans: A Feedback: The fetus is in a transverse lie with the shoulder as the presenting part, necessitating a cesarean birth. Vaginal birth, forceps-assisted, and vacuum extraction births are not appropriate.

Assessment of a woman in labor reveals cervical dilation of 3 cm, cervical effacement of 30%, and contractions occurring every 7 to 8 minutes, lasting about 40 seconds. The nurse determines that this client is in: A) Latent phase of the first stage B) Active phase of the first stage C) Transition phase of the first stage D) Perineal phase of the second stage

Ans: A Feedback: The latent phase of the first stage of labor involves cervical dilation of 0 to 3 cm, cervical effacement of 0% to 40%, and contractions every 5 to 10 minutes lasting 30 to 45 seconds. The active phase is characterized by cervical dilation of 4 to 7 cm, effacement of 40% to 80%, and contractions occurring every 2 to 5 minutes lasting 45 to 60 seconds. The transition phase is characterized by cervical dilation of 8 to 10 cm, effacement of 80% to 100%, and contractions occurring every 1 to 2 minutes lasting 60 to 90 seconds. The perineal phase of the second stage occurs with complete cervical dilation and effacement, contractions occurring every 2 to 3 minutes and lasting 60 to 90 seconds, and a tremendous urge to push by the mother.

Which position would be most appropriate for the nurse to suggest as a comfort measure to a woman who is in the first stage of labor? (Select all that apply.) (3) A) Walking with partner support B) Straddling with forward leaning over a chair C) Closed knee-chest position D) Rocking back and forth with foot on chair E) Supine with legs raised at a 90-degree angle

Ans: A, B, D Positioning during the first stage of labor includes walking with support from the partner, side-lying with pillows between the knees, leaning forward by straddling a chair, table, or bed or kneeling over a birthing ball, lunging by rocking weight back and forth with a foot up on a chair or birthing ball or an open knee-chest position.(pg. 475)

After teaching a group of students about the factors affecting the labor process, the instructor determines that the teaching was successful when the group identifies which of the following as a component of the true pelvis? (Select all that apply.) (3) A) Pelvic inlet B) Cervix C) Mid pelvis D) Pelvic outlet E) Vagina F) Pelvic floor muscles

Ans: A, C, D Feedback: The true pelvis is made up of three planes: the pelvic inlet, mid pelvis, and pelvic outlet. The cervix, vagina, and pelvic floor muscles are the soft tissues of the passageway.

A nurse is preparing a presentation for a group of pregnant women about the labor experience. Which of the following would the nurse most likely include when discussing measures to promote coping for a positive labor experience? (Select all that apply.) (3) A) Presence of a support partner B) View of birth as a stressor C) Low anxiety level D) Fear of loss of control E) Participation in a pregnancy exercise program

Ans: A, C, E Feedback: Numerous factors can affect a woman's coping ability during labor and birth. Having the presence and support of a valued partner during labor, engaging in exercise during pregnancy, viewing the birthing experience as a meaningful rather than stressful event, and a low anxiety level can promote a woman's ability to cope. Excessive anxiety may interfere with the labor progress, and fear of labor and loss of control may enhance pain perception, increasing the fear.

A nurse is preparing a class for pregnant women about labor and birth. When describing the typical movements that the fetus goes through as it travels through the passageway, which of the following would the nurse most likely include? (Select all that apply.) (3) A) Internal rotation B) Abduction C) Descent D) Pronation E) Flexion

Ans: A, C, E Feedback: The positional changes that occur as the fetus moves through the passageway are called the cardinal movements of labor and include engagement, descent, flexion, internal rotation, extension, external rotation, and expulsion. The fetus does not undergo abduction or pronation.

A woman in her third trimester comes to the clinic for a prenatal visit. During assessment the woman reports that her breathing has become much easier in the last week but she has noticed increased pelvic pressure , cramping and lower back pain. The nurse determines that which of the following has most likely occurred? A) Cervical dilation B) Lightening C) Bloody show D) Braxton-Hicks contractions

Ans: B Feedback: Lightening occurs when the fetal presenting part begins to descend into the maternal pelvis. The uterus lowers and moves into the maternal pelvis. The shape of the abdomen changes as a result of the change in the uterus. The woman usually notes that her breathing is much easier. However, she may complain of increased pelvic pressure, cramping, and lower back pain. Although cervical dilation also may be occurring, it does not account for the woman's complaints. Bloody show refers to passage of the mucous plug that fills the cervical canal during pregnancy. It occurs with the onset of labor. Braxton-Hicks contractions increase in strength and frequency and aid in moving the cervix from a posterior position to an anterior position. They also help in ripening and softening the cervix.

A woman calls the health care facility stating that she is in labor. The nurse would urge the client to come to the facility if the client reports which of the following? A) Increased energy level with alternating strong and weak contractions B) Moderately strong contractions every 4 minutes, lasting about 1 minute C) Contractions noted in the front of abdomen that stop when she walks D) Pink-tinged vaginal secretions and irregular contractions lasting about 30 seconds

Ans: B Feedback: Moderately strong regular contractions 60 seconds in duration indicate that the client is probably in the active phase of the first stage of labor. Alternating strong and weak contractions, contractions in the front of the abdomen that change with activity, and pink-tinged secretions with irregular contractions suggest false labor.

A client is admitted to the labor and birthing suite in early labor. On review of her medical record, the nurse determines that the client's pelvic shape as identified in the antepartal progress notes is the most favorable one for a vaginal delivery. Which pelvic shape would the nurse have noted? A) Platypelloid B) Gynecoid C) Android D) Anthropoid

Ans: B Feedback: The most favorable pelvic shape for vaginal delivery is the gynecoid shape. The anthropoid pelvis is favorable for vaginal birth but it is not the most favorable shape. The android pelvis is not considered favorable for a vaginal birth because descent of the fetal head is slow and failure of the fetus to rotate is common. Women with a platypelloid pelvis usually require cesarean birth.

When describing the stages of labor to a pregnant woman, which of the following would the nurse identify as the major change occurring during the first stage? A) Regular contractions B) Cervical dilation C) Fetal movement through the birth canal D) Placental separation

Ans: B Feedback: The primary change occurring during the first stage of labor is progressive cervical dilation. Contractions occur during the first and second stages of labor. Fetal movement through the birth canal is the major change during the second stage of labor. Placental separation occurs during the third stage of labor.

A woman telephones her health care provider and reports that her "water just broke." Which suggestion by the nurse would be most appropriate? A) "Call us back when you start having contractions." B) "Come to the clinic or emergency department for an evaluation." C) "Drink 3 to 4 glasses of water and lie down." D) "Come in as soon as you feel the urge to push."

Ans: B Feedback: When the amniotic sac ruptures, the barrier to infection is gone and there is the danger of cord prolapse if engagement has not occurred. Therefore, the nurse should suggest that the woman come in for an evaluation. Calling back when contractions start, drinking water, and lying down are inappropriate because of the increased risk for infection and cord prolapse. Telling the client to wait until she feels the urge to push is inappropriate because this occurs during the second stage of labor.

When assessing cervical effacement of a client in labor, the nurse assesses which of the following characteristics? A) Extent of opening to its widest diameter B) Degree of thinning C) Passage of the mucous plug D) Fetal presenting part

Ans: B Feedback: Effacement refers to the degree of thinning of the cervix. Cervical dilation refers to the extent of opening at the widest diameter. Passage of the mucous plug occurs with bloody show is a premonitory sign of labor. The fetal presenting part is determined by vaginal examination and is commonly the head (cephalic), pelvis (breech), or shoulder.

1. A woman in her 40th week of pregnancy calls the nurse at the clinic and says she's not sure whether she is in true or false labor. Which statement by the client would lead the nurse to suspect that the woman is experiencing false labor? A) "I'm feeling contractions mostly in my back." B) "My contractions are about 6 minutes apart and regular." C) "The contractions slow down when I walk around." D) "If I try to talk to my partner during a contraction, I can't."

Ans: C Feedback: False labor is characterized by contractions that are irregular and weak, often slowing down with walking or a position change. True labor contractions begin in the back and radiate around toward the front of the abdomen. They are regular and become stronger over time; the woman may find it extremely difficult if not impossible to have a conversation during a contraction.

A nurse is documenting fetal lie of a woman in labor. Which term would the nurse most likely use? A) Flexion B) Extension C) Longitudinal D) Cephalic

Ans: C Feedback: Fetal lie refers to the relationships of the long axis (spine) of the fetus to the long axis (spine) of the mother. There are two primary lies: longitudinal and transverse. Flexion and extension are terms used to describe fetal attitude. Cephalic is a term used to describe fetal presentation.

Assessment of a pregnant woman reveals that the presenting part of the fetus is at the level of the maternal ischial spines. The nurse documents this as which station? A) -2 B) -1 C) 0 D) +1

Ans: C Feedback: Station refers to the relationship of the presenting part to the level of the maternal pelvic ischial spines. Fetal station is measured in centimeters and is referred to as a minus or plus, depending on its location above or below the ischial spines. Zero (0) station is designated when the presenting part is at the level of the maternal ischial spines. When the presenting part is above the ischial spines, the distance is recorded as minus stations. When the presenting part is below the ischial spines, the distance is recorded as plus stations

A client has not received any medication during her labor. She is having frequent contractions every 1 to 2 minutes and has become irritable with her coach and no longer will allow the nurse to palpate her fundus during contractions. Her cervix is 8 cm dilated and 90% effaced. The nurse interprets these findings as indicating: A) Latent phase of the first stage of labor B) Active phase of the first stage of labor C) Transition phase of the first stage of labor D) Pelvic phase of the second stage of labor

Ans: C Feedback: The transition phase is characterized by cervical dilation of 8 to 10 cm, effacement of 80% to 100%, contractions that are strong, painful, and frequent (every 1 to 2 minutes) and last 60 to 90 seconds, and irritability, apprehension, and feelings of loss of control. The latent phase is characterized by mild contractions every 5 to 10 minutes, cervical dilation of 0 to 3 cm and effacement of 0% to 40%, and excitement and frequent talking by the mother. The active phase is characterized by moderate to strong contractions every 2 to 5 minutes, cervical dilation of 4 to 7 cm and effacement of 40% to 80%, with the mother becoming intense and inwardly focused. The pelvic phase of the second stage of labor is characterized by complete cervical dilation and effacement, with strong contractions every 2 to 3 minutes; the mother focuses on pushing.

A woman is in the first stage of labor. The nurse would encourage her to assume which position to facilitate the progress of labor? A) Supine B) Lithotomy C) Upright D) Knee-chest

Ans: C Feedback: The use of any upright position helps to reduce the length of labor. Research validates that nonmoving back-lying positions such as supine and lithotomy positions during labor are not healthy. The knee-chest position would assist in rotating the fetus in a posterior position.

A nurse is assessing a woman in labor. Which finding would the nurse identify as a cause for concern during a contraction? A) Heart rate increase from 76 bpm to 90 bpm B) Blood pressure rise from 110/60 mm Hg to 120/74 C) White blood cell count of 12,000 cells/mm3 D) Respiratory rate of 10 breaths /minute

Ans: D Feedback: During labor, the mother experiences various physiologic responses including an increase in heart rate by 10 to 20 bpm, a rise in blood pressure by up to 35 mm Hg during a contraction, an increase in white blood cell count to 25,000 to 30,000 cells/mm3, perhaps as a result of tissue trauma, and an increase in respiratory rate with greater oxygen consumption due to the increase in metabolism. A drop in respiratory rate would be a cause for concern.

A nurse is caring for several women in labor. The nurse determines that which woman is in the transition phase of labor? A) Contractions every 5 minutes, cervical dilation 3 cm B) Contractions every 3 minutes, cervical dilation 5 cm C) Contractions every 2½ minutes, cervical dilation 7 cm D) Contractions every 1 minute, cervical dilation 9 cm

Ans: D Feedback: The transition phase is characterized by strong contractions occurring every 1 to 2 minutes and cervical dilation from 8 to 10 cm. Contractions every 5 minutes with cervical dilation of 3 cm is typical of the latent phase. Contractions every 3 minutes with cervical dilation of 5 cm and contractions every 21/2 minutes with cervical dilation of 7 cm suggest the active phase of labor.

A fetus is assessed at 2 cm above the ischial spines. The nurse would document fetal station as: A) +4 B) +2 C) 0 D) -2

Ans: D Feedback: When the presenting part is above the ischial spines, it is noted as a negative station. Since the measurement is 2 cm, the station would be -2. A 0 station indicates that the fetal presenting part is at the level of the ischial spines. Positive stations indicate that the presenting part is below the level of the ischial spines.

The nurse caring for a postpartum client explains the occurrence of lochia following delivery. Which of the following statements accurately describe a characteristic of this process? A) "Lochia ruba, which is mostly red and bloody, is seen for the first week." B) "Lochia serosa, which is pink or brown tinged, starts after the bleeding diminishes." C) "Lochia alba, which is yellow or white, starts on about day 15. D) "Lochia alba has a pungent, foul odor."

B) "Lochia serosa, which is pink or brown tinged, starts after the bleeding diminishes."

Which of the following would the nurse assess in a client experiencing abruptio placenta? A. Bright red, painless vaginal bleeding B. Concealed or external dark red bleeding C. Palpable fetal outline D. Soft and nontender abdomen

B. A client with abruptio placentae may exhibit concealed or dark red bleeding, possibly reporting sudden intense localized uterine pain. The uterus is typically firm to boardlike, and the fetal presenting part may be engaged. Bright red, painless vaginal bleeding, a palpable fetal outline and a soft nontender abdomen are manifestations of placenta previa

During which of the following stages of labor would the nurse assess "crowning"? a. First stage b. Second stage c. Third stage d. Fourth stage

B. Crowing, which occurs when the newborn's head or presenting part appears at the vaginal opening, occurs during the second stage of labor. During the first stage of labor, cervical dilation and effacement occur. During the third stage of labor, the newborn and placenta are delivered. The fourth stage of labor lasts from 1 to 4 hours after birth, during which time the mother and newborn recover from the physical process of birth and the mother's organs undergo the initial readjustment to the nonpregnant state.

When PROM occurs, which of the following provides evidence of the nurse's understanding of the client's immediate needs? A. The chorion and amnion rupture 4 hours before the onset of labor. B. PROM removes the fetus most effective defense against infection C. Nursing care is based on fetal viability and gestational age. D. PROM is associated with malpresentation and possibly incompetent cervix

B. PROM can precipitate many potential and actual problems; one of the most serious is the fetus loss of an effective defense against infection. This is the client's most immediate need at this time. Typically, PROM occurs about 1 hour, not 4 hours, before labor begins. Fetal viability and gestational age are less immediate considerations that affect the plan of care. Malpresentation and an incompetent cervix may be causes of PROM.

Which of the following is the nurse's initial action when umbilical cord prolapse occurs? A. Begin monitoring maternal vital signs and FHR B. Place the client in a knee-chest position in bed C. Notify the physician and prepare the client for delivery D. Apply a sterile warm saline dressing to the exposed cord

B. The immediate priority is to minimize pressure on the cord. Thus the nurse's initial action involves placing the client on bed rest and then placing the client in a knee-chest position or lowering the head of the bed, and elevating the maternal hips on a pillow to minimize the pressure on the cord. Monitoring maternal vital signs and FHR, notifying the physician and preparing the client for delivery, and wrapping the cord with sterile saline soaked warm gauze are important. But these actions have no effect on minimizing the pressure on the cord.

Which of the following may happen if the uterus becomes overstimulated by oxytocin during the induction of labor? A. Weak contraction prolonged to more than 70 seconds B. Tetanic contractions prolonged to more than 90 seconds C. Increased pain with bright red vaginal bleeding D. Increased restlessness and anxiety

B. Tetanic contractions prolonged to more than 90 seconds

A pregnant client in her 38th week of gestation complains of abdominal pains and suspects she is in labor. Which of the following findings are characteristic of true labor contractions? A) Contractions are short and irregular. B) Contractions are generally felt low in the abdomen. C) Contractions help create effacement and dilation of the cervix. D) Contractions are relieved by change of position or activity.

C) Contractions help create effacement and dilation of the cervix.

A nurse is monitoring the fetal heart rate of a client. What signs of fetal distress on the fetal monitor should the nurse report immediately? A) Accelerations of 15 bpm B) Early decelerations C) Decreased variability D) Fetal heart rate above 100 bpm

C) Decreased variability

The nurse assessing a laboring client documents that the client is in stage II of labor. What typically occurs during this stage? A) Dilation of the cervix B) Movement of fetus to the birth canal C) Delivery of the baby through the vaginal opening D) Delivery of the placenta following delivery of the newborn

C) Delivery of the baby through the vaginal opening

The nurse is caring for a client who just delivered a healthy 7 pound 9 ounce baby girl. What is the priority nursing intervention for this new mother? A) Give the mother a bed bath. B) Allow the mother a period of rest with no distractions. C) Encourage bonding with the infant. D) Perform a comprehensive assessment.

C) Encourage bonding with the infant.

When preparing a client for cesarean delivery, which of the following key concepts should be considered when implementing nursing care? A. Instruct the mother's support person to remain in the family lounge until after the delivery B. Arrange for a staff member of the anesthesia department to explain what to expect postoperatively C. Modify preoperative teaching to meet the needs of either a planned or emergency cesarean birth D. Explain the surgery, expected outcome, and kind of anesthetics

C. A key point to consider when preparing the client for a cesarean delivery is to modify the preoperative teaching to meet the needs of either a planned or emergency cesarean birth, the depth and breadth of instruction will depend on circumstances and time available. Allowing the mother's support person to remain with her as much as possible is an important concept, although doing so depends on many variables. Arranging for necessary explanations by various staff members to be involved with the client's care is a nursing responsibility. The nurse is responsible for reinforcing the explanations about the surgery, expected outcome, and type of anesthetic to be used. The obstetrician is responsible for explaining about the surgery and outcome and the anesthesiology staff is responsible for explanations about the type of anesthesia to be used.

Barbiturates are usually not given for pain relief during active labor for which of the following reasons? A. The neonatal effects include hypotonia, hypothermia, generalized drowsiness, and reluctance to feed for the first few days. B. These drugs readily cross the placental barrier, causing depressive effects in the newborn 2 to 3 hours after intramuscular injection. C. They rapidly transfer across the placenta, and lack of an antagonist make them generally inappropriate during labor. D. Adverse reactions may include maternal hypotension, allergic or toxic reaction or partial or total respiratory failure

C. Barbiturates are rapidly transferred across the placental barrier, and lack of an antagonist makes them generally inappropriate during active labor. Neonatal side effects of barbiturates include central nervous system depression, prolonged drowsiness, delayed establishment of feeding (e.g. due to poor sucking reflex or poor sucking pressure). Tranquilizers are associated with neonatal effects such as hypotonia, hypothermia, generalized drowsiness, and reluctance to feed for the first few days. Narcotic analgesic readily cross the placental barrier, causing depressive effects in the newborn 2 to 3 hours after intramuscular injection. Regional anesthesia is associated with adverse reactions such as maternal hypotension, allergic or toxic reaction, or partial or total respiratory failure.

A primigravida in active labor is about 9 days post-term. The client desires a bilateral pudendal block anesthesia before delivery. After the nurse explains this type of anesthesia to the client, which of the following locations identified by the client as the area of relief would indicate to the nurse that the teaching was effective? A. Back B. Abdomen C. Fundus D. Perineum

Correct Answer: D. Perineum A bilateral pudendal block is used for vaginal deliveries to relieve pain primarily in the perineum and vagina. Pudendal block anesthesia is adequate for episiotomy and its repair. Option A: A spinal anesthetic is given into the middle of the lower back and local anesthetic is injected through the needle into the fluid that surrounds the spinal cord. It numbs the nerves that supply the abdomen, hips, bottom, and legs. Option B: General or regional anesthesia can be appropriate for patients undergoing abdominal surgery. Balanced anesthesia with inhalational anesthetics, opioids, and neuromuscular blockers are used in general anesthesia for abdominal surgical procedures. Option C: Spinal anesthesia is one of the most preferred anesthetic methods during Cesarean section since it provides easy and rapid induction, effective sensory and motor block, and has little effect on the fetus.

A pregnant client is diagnosed with partial placenta previa. In explaining the diagnosis, the nurse tells the client that the usual treatment for partial placenta previa is which of the following? A. Activity limited to bed rest. B. Platelet infusion. C. Immediate cesarean delivery. D. Labor induction with oxytocin.

Correct Answer: A. Activity limited to bed rest Treatment of partial placenta previa includes bed rest, hydration, and careful monitoring of the client's bleeding. Option B: The greatest risk of placenta previa is hemorrhage. Bleeding often occurs as the lower part of the uterus thins during the third trimester of pregnancy in preparation for labor. This may require blood transfusion during Cesarean section. Option C: In general, there is a higher Cesarean rate associated with placental edge-to-cervical os distances of less than 2 cm. Option D: Labor induction is the stimulation of uterine contractions during pregnancy before labor begins on its own to achieve a vaginal birth. It is not an option for placenta previa.

23. Which of the following nursing interventions would the nurse perform during the third stage of labor? A. Obtain a urine specimen and other laboratory tests. B. Assess uterine contractions every 30 minutes. C. Coach for effective client pushing D. Promote parent-newborn interaction.

D. During the third stage of labor, which begins with the delivery of the newborn, the nurse would promote parent-newborn interaction by placing the newborn on the mother's abdomen and encouraging the parents to touch the newborn. Collecting a urine specimen and other laboratory tests is done on admission during the first stage of labor. Assessing uterine contractions every 30 minutes is performed during the latent phase of the first stage of labor. Coaching the client to push effectively is appropriate during the second stage of labor.

Immediately before expulsion, which of the following cardinal movements occur? A. Descent B. Flexion C. Extension D. External rotation

D. Immediately before expulsion or birth of the rest of the body, the cardinal movement of external rotation occurs. Descent flexion, internal rotation, extension, and restitution (in this order) occur before external rotation

FHR can be auscultated with a fetoscope as early as which of the following? a. 5 weeks gestation b. 10 weeks gestation c. 15 weeks gestation d. 20 weeks gestation

D. The FHR can be auscultated with a fetoscope at about 20 week's gestation. FHR usually is ausculatated at the midline suprapubic region with Doppler ultrasound transducer at 10 to 12 week's gestation. FHR, cannot be heard any earlier than 10 weeks' gestation.

Which of the following amounts of blood loss following birth marks the criterion for describing postpartum hemorrhage? A. More than 200 ml B. More than 300 ml C. More than 400 ml D. More than 500 ml

D. More than 500 ml

One hour postdelivery the nurse notes the new mother has saturated three perineal pads. What is the most appropriate nursing action? a. Check the fundus for position and firmness. b. Report to the doctor immediately. c. Change the pads and chart the time. d. Time how long it takes to soak one pad.

a. Check the fundus for position and firmness.

A woman in the active phase of the first stage of labor is using a shallow pattern of breathing, which is about twice the normal adult breathing rate. She starts to complain about feeling lightheaded and dizzy and states that her fingers are tingling. The nurse should: a. Notify the womans physician. b. Tell the woman to slow the pace of her breathing. c. Administer oxygen via a mask or nasal cannula. d. Help her breathe into a paper bag

ans: d This woman is experiencing the side effects of hyperventilation, which include the symptoms of lightheadedness, dizziness, tingling of the fingers, or circumoral numbness. Having the woman breathe into a paper bag held tightly around her mouth and nose may eliminate respiratory alkalosis. This enables her to rebreathe carbon dioxide and replace the bicarbonate ion.

Which assessment finding is most important as labor progresses? a. Labor is completed within 18 hours. b. The pulse and respirations rise with the work of labor. c. The uterus relaxes completely between contractions. d. The client is remaining in control of emotions.

c. The uterus relaxes completely between contractions.

A nurse performs an initial assessment of a laboring woman and reports the following findings to the primary care provider: fetal heart rate is 152 bpm, cervix is 100% effaced and 5 cm dilated, membranes are intact, and presenting part is well applied to the cervix and at -1 station. The nurse recognizes that the client is in which stage of labor? a. first, latent b. third c. first, active d. second

c. first, active

In documenting labor experiences, nurses should know that a uterine contraction is described according to all of these characteristics except: a) Frequency (how often contractions occur) b) Intensity (the strength of the contraction at its peak) c) Resting tone (The tension in the uterine muscle) d) Appearance (shape and height)

d) Appearance (shape and height)

The most common cause of decreased variability in the FHR that lasts 30 minutes or less is: a) Altered cerebral blood flow b) Fetal hypoxemia c) Umbilical cord compression d) Fetal sleep cycles

d) Fetal sleep cycles

A nurse in the delivery room is assisting with the delivery of a newborn infant. After the delivery of the newborn, the nurse assists in delivering the placenta. Which observation would indicate that the placenta has separated from the uterine wall and is ready for delivery? A. The umbilical cord shortens in length and changes in color B. A soft and boggy uterus C. Maternal complaints of severe uterine cramping d. Changes in the shape of the uterus

d. Changes in the shape of the uterus Explanation: Signs of placental separation include lengthening of the umbilical cord, a sudden gush of dark blood from the introitus (vagina), a firmly contracted uterus, and the uterus changing from a discoid (like a disk) to a globular (like a globe) shape. The client may experience vaginal fullness, but not severe uterine cramping.

The nurse is assessing a client who has given birth within the past hour. The nurse would expect to find the woman's fundus at which location? a. one finger breadth below the umbilicus b. between the umbilicus and symphysis pubis c. 2 cm above the umbilicus d. at the level of the umbilicus

d. at the level of the umbilicus


Set pelajaran terkait

Psychiatric-Mental Health Practice Exam HESI#2

View Set

Transitions to use on the final!

View Set

REAL ESTATE FINANCE JUNE 24 (73)

View Set

Smartbook 22 : Immune Systme and the Body's Defense

View Set